Peds Exam 1

Ace your homework & exams now with Quizwiz!

A 3-year-old is brought to the ER with coughing and gagging. The parent reports that the child was eating carrots when she began to gag. Which diagnostic evaluation will be used to determine if the child has aspirated carrots? 1. Chest x-ray. 2. Bronchoscopy. 3. Arterial blood gas (ABG). 4. Sputum culture.

2. A bronchoscopy will allow the physician to visualize the airway and will help determine if the child aspirated the carrot. TEST-TAKING HINT: Answer 1 can be eliminated because items that are not radiopaque (opaque to x-rays) cannot be seen on an x-ray. Answers 3 and 4 can be eliminated because they do not provide confirmation regarding whether the child aspirated.

Which would be appropriate nursing care management of a child with the diagnosis of mononucleosis? 1. Only family visitors. 2. Bedrest. 3. Clear liquids. 4. Limited daily fluid intake.

26. 1. Children with mononucleosis are more susceptible to secondary infections. Therefore, they should be limited to visitors within the family, especially during the acute phase of illness. TEST-TAKING HINT: The test taker can eliminate answers 2 and 3 by understanding mononucleosis. Children with mononucleosis are usually very tired, are not interested in engaging in vigorous activity, and are rarely interested in eating.

The parent of an infant with cystic fibrosis (CF) asks the nurse how to meet the child's increased nutritional needs. Which is the nurse's best suggestion? 1. "You may need to increase the number of fresh fruits and vegetables you give your child." 2. "You may need to advance your child's diet to whole cow's milk because it is higher in fat than formula." 3. "You may need to change your child to a higher-calorie formula." 4. "You may need to increase your child's carbohydrate intake."

3. Often infants with CF need to have a higher-calorie formula to meet their nutritional needs. Infants may also be placed on hydrolysate formulas that contain medium-chain triglycerides. TEST-TAKING HINT: Answers 1, 2, and 4 can be eliminated with understanding of the nutritional needs of the child with CF. Answer 2 can also be eliminated because whole cow's milk is not recommended until 12 months of age.

The nurse is reviewing discharge instructions with the parents of a child who had a tonsillectomy 24 hours ago. The parents tell the nurse that the child is a big eater, and they want to know what foods to give the child for the next 24 hours. What is the nurse's best response? 1. "The child's diet should not be restricted at all." 2. "The child's diet should be restricted to clear liquids." 3. "The child's diet should be restricted to ice cream and cold liquids." 4. "The child's diet should be restricted to soft foods."

4. Soft foods are recommended to limit the child's pain and to decrease the risk for bleeding. TEST-TAKING HINT: The test taker can eliminate answer 1 by knowing there are usually some dietary restrictions following any surgical procedure.

8. The nursing student studying respiratory anatomy and physiology learns that which tissue or organ grows faster than any other tissue or organ in a child? A. Diaphragm B. Epiglottis C. Lungs D. Tonsils

ANS: D The lymphoid tissue of the tonsils is absent at birth, but grows more rapidly in a child than any other tissue.

A school-age child has been diagnosed with nasopharyngitis. The parent is concerned because the child has had little or no appetite for the last 24 hours. Which is the nurse's best response? 1. "Do not be concerned; it is common for children to have a decreased appetite during a respiratory illness." 2. "Be sure your child is taking an adequate amount of fluids. The appetite should return soon." 3. "Try offering the child some favorite food. Maybe that will improve the appetite." 4. "You need to force your child to eat whatever you can; adequate nutrition is essential."

2. It is common for children to have a decreased appetite when they have a respiratory illness. The nurse is appropriately instructing the parent that the child will be fine by taking in an adequate amount of fluid. TEST-TAKING HINT: Answer 4 can be eliminated because one should not force thechild to eat. If the word had been "encourage," it would have been a better choice, although still not the best answer. Answer 1 can be eliminated because the nurse did not inform the parent of the importance of maintaining adequate fluid intake.

Which position would be most comfortable for a child with left-sided pneumonia? 1. Trendelenburg. 2. Left side. 3. Right side. 4. Supine.

2. Lying on the left side may provide the patient with the most comfort. Lying on the left splints the chest and reduces the pleural rubbing. TEST-TAKING HINT: The test taker can eliminate answers 1 and 4 because neither of them would improve the child's respiratory effort. Both these positions may actually cause the patient increased respiratory distress.

21. Which foods would the nurse recommend to the mother of a 2-year-old with anemia? 1. 32 oz of whole cow's milk per day. 2. Meats, eggs, and green vegetables. 3. Fruits, whole grains, and rice. 4. 8 oz of juice, three times per day.

2. Meats, eggs, and green vegetables.

8. Which toy is the best choice for a 12-month-old? 1. Baby doll. 2. Musical rattle. 3. Board book. 4. Colorful beads.

2. Musical rattle. A musical rattle is the perfect toy for this child. Infants have short attention spans and enjoy auditory and visual stimulation.

Which would be an early sign of respiratory distress in a 2-month-old? 1. Breathing shallowly. 2. Tachypnea. 3. Tachycardia. 4. Bradycardia.

2. Tachypnea is an early sign of distress and is often the first sign of respiratory illness in infants TEST-TAKING HINT: The test taker must know the signs and symptoms of respiratory distress and be able to recognize them

31. Which reaction would a nurse expect when giving a preschooler immunizations? 1. The child remains silent and still. 2. The child cries and tells the nurse that it hurts. 3. The child tries to stall the nurse. 4. The child remains still while telling the nurse that she is hurting him.

2. The child cries and tells the nurse that it hurts. The common response of a 5-year-old is to cry and protest during an immunization. School-age children are most likely to try to stall the nurse. Teens usually remain still, and they may calmly tell the nurse that they are feeling pain during the injection.

A 5-year-old is brought to the ER with a temperature of 99.5°F (37.5°C), a barky cough, stridor, and hoarseness. Which nursing intervention should the nurse prepare for? 1. Immediate IV placement. 2. Respiratory treatment of racemic epinephrine. 3. A tracheostomy set at the bedside. 4. Informing the child's parents about a tonsillectomy.

2. The child has stridor, indicating airway edema, which can be relieved by aerosolized racemic epinephrine. TEST-TAKING HINT: The test taker must accurately identify that the question is describing a child with croup and know the accepted treatments.

4. Which statement by an infant's mother leads the nurse to believe that she needs further education about the nutritional needs of a 6-month-old? 1. "I will continue to breastfeed my son and will give him rice cereal three times a day." 2. "I will start my son on fruits and gradually introduce vegetables." 3. "I will start my son on carrots and will introduce one new vegetable every few days." 4. "I will not give my son any more than 8 ounces of baby juice per day."

2. "I will start my son on fruits and gradually introduce vegetables." Infants should be started on vegetables prior to fruits. The sweetness of fruits may inhibit infants from taking vegetables. Cereal can be introduced between 4 and 6 months of age. Infants can be given fruit juice by 6 months of age, but it is recommended not to exceed 4 to 6 ounces per day.

10. The mother of a newborn asks the nurse when the infant will receive the first hepatitis B immunization. Which is the nurse's best response? 1. "Babies receive the hepatitis B vaccine only if their mother is hepatitis B-positive." 2. "The first dose of the hepatitis B vaccine will be given prior to discharge today." 3. "The first dose of hepatitis B vaccine is given at 1 year of age." 4. "Babies receive their first hepatitis B vaccine at 6 months of age."

2. "The first dose of the hepatitis B vaccine will be given prior to discharge today." The first dose of hepatitis B vaccine is recommended between birth and 2 months. In most hospitals, newborns are given the vaccine prior to discharge.

Which child would benefit most from having ear tubes placed? 1. A 2-month-old who has had one ear infection. 2. A 2-year-old who has had five previous ear infections. 3. A 3-year-old whose sibling has had four ear infections. 4. A 7-year-old who has had two ear infections this year.

2. A 2-year-old who has had multiple ear infections is a perfect candidate for ear tubes. The other issue is that a 2-year-old is at the height of language development, which can be adversely affected by recurrent ear infections. TEST-TAKING HINT: The test taker must also consider the developmental level of the child in this question. The 2-year-old has had multiple infections and is also at a stage when language development is essential. If this child is not hearing appropriately, speech will also be delayed. Surgical intervention is reserved for those who have had recurrent infections.

Which should be included in instructions to the parent of a child prescribed amoxicillin to treat an ear infection? 1. "Continue the amoxicillin until the child's symptoms subside." 2. "Administer an over-the-counter antihistamine with the antibiotic." 3. "Administer the amoxicillin until all the medication is gone." 4. "Allow your child to administer his own dose of amoxicillin."

3. It is essential that all the medication be given. TEST-TAKING HINT: Answer 1 can be eliminated because a course of antibiotics should always be completed as ordered, no matter what the age of the child. Answer 4 can be eliminated because children would not be expected to administer their own medications without supervision by an adult.

Which should the nurse instruct children to do to stop the spread of influenza in the classroom? 1. Stay home if they have a runny nose and cough. 2. Wash their hands after using the restroom. 3. Wash their hands after sneezing. 4. Have a flu shot annually.

3. It is essential that children wash their hands after any contact with nasopharyngeal secretions. TEST-TAKING HINT: Answers 1 and 4 can be eliminated because both situations are under parental control.

What information should the nurse provide the parent of a child diagnosed with nasopharyngitis? 1. Complete the entire prescription of antibiotics. 2. Avoid sending the child to day care. 3. Use comfort measures for the child. 4. Restrict the child to clear liquids for 24 hours.

3. Nursing care for nasopharyngitis is primarily supportive. Keeping the child comfortable during the course of the illness is all the parents can do. Nasal congestion can be relieved using normal saline drops and bulb suction. Tylenol can also be given for discomfort or a mild fever. TEST-TAKING HINT: This question requires the test taker to understand how nasopharyngitis is treated.

43. Which is the best method of distraction for an 8-year-old who is having surgery later today and is NPO? 1. Use the telephone to call friends. 2. Watch television. 3. Play a board game. 4. Read the central line pamphlet he was given.

3. Play a board game. A board game is the optimal choice be- cause school-age children enjoy being engaged in an activity with others that will require some skill and challenge.

20. The mother of a child 2 years 6 months has arranged a play date with the neighbor and her child 2 years 9 months. During the play date the two mothers should expect that the children will do which of the following? 1. Share and trade their toys while playing. 2. Play with one another with little or no conflict. 3. Play alongside one another but not actively with one another. 4. Only play with one or two items, ignoring most of the other toys.

3. Play alongside one another but not actively with one another. Toddlers engage in parallel play. They often play alongside another child, but they rarely engage in activities with the other child. Toddlers do not share their possessions well. One of their favorite words is "mine."

40. Which nursing action is most appropriate to gain information about how a child is feeling? 1. Actively attempt to make friends with the child before asking about her feelings. 2. Ask the child's parents what feelings she has expressed in regard to her diagnosis. 3. Provide the child with some paper to draw a picture of how she is feeling. 4. Ask the child direct questions about how she is feeling.

3. Provide the child with some paper to draw a picture of how she is feeling. Often children will include much more detail about their feelings in drawings. They will often express things in pic- tures they are unable to verbalize.

35. A 3-year-old admitted to the hospital with croup has the following vital signs: heart rate 90, respiratory rate 44, blood pressure 100/52, and temperature 98.8°F (37.1°C). The parents ask the nurse if these vital signs are normal. The nurse's best response is: 1. "Your son's blood pressure is elevated, but the other vital signs are within the normal range.." 2. "Your son's temperature is elevated, but the other vital signs are within the normal range.." 3. "Your son's respiratory rate is elevated, but the other vital signs are within the normal range." 4. "Your son's heart rate is elevated, but the other vital signs are within the normal range."

3. "Your son's respiratory rate is elevated, but the other vital signs are within the normal range." A normal respiratory rate for a child from 3 to 6 years is 20 to 30 breaths per minute.

A parent asks the nurse what will need to be done to relieve the constipation of her child who also has cystic fibrosis (CF). Which is the nurse's best response? 1. "Your child likely has an obstruction and will require surgery." 2. "Your child will likely be given IV fluids." 3. "Your child will likely be given MiraLAX." 4. "Your child will be placed on a clear liquid diet."

3. CF patients with constipation commonly receive a stool softener or an osmotic solution such as polyethylene glycol 3350 (MiraLAX) orally to relieve their constipation. TEST-TAKING HINT: Answer 1 can be eliminated because surgery is not indicated for constipation.

28. The best method to explain a procedure to a hospitalized preschool-age child is to: 1. Show the child a pamphlet with pictures showing the procedure. 2. Have the 5-year-old next door tell the 4-year-old about the experience. 3. Demonstrate the procedure on a doll. 4. Show the child a video of the procedure

3. Demonstrate the procedure on a doll. A 4-year-old child understands in very concrete and simple terms. Therefore, medical play is an excellent method for helping to understand the procedure.

2. How can the nurse best facilitate the trust relationship between infant and parents while the infant is hospitalized? The nurse should: 1. Encourage the parents to remain at their child's bedside as much as possible. 2. Keep parents informed about all aspects of their child's condition. 3. Encourage the parents to hold their child as much as possible. 4. Advise the parents to participate actively in their child's care.

3. Encourage the parents to hold their child as much as possible. Having parents hold their child while in the hospital is an excellent means of building the trust relationship. Infants are most secure when they are being held, patted, and spoken to.

23. Which should the nurse do to prevent separation anxiety in a hospitalized toddler? 1. Assume the parental role when parents are not able to be at the bedside. 2. Encourage the parents to always remain at the bedside. 3. Establish a routine similar to that of the child's home. 4. Rotate nursing staff so the child becomes comfortable with a variety of nurses.

3. Establish a routine similar to that of the child's home. It is very important to try to maintain a child's home routine both when par- ents are present and when they have to leave the hospital. This will increase the child's sense of security and decrease anxiety. Providing consistent nursing care is impor- tant, not rotating staff.

A 7-month-old has a low-grade fever, nasal congestion, and a mild cough. What should the nursing care management of this child include? 1. Maintaining strict bedrest. 2. Avoiding contact with family members. 3. Instilling saline nose drops and bulb suctioning. 4. Keeping the head of the bed flat.

3. Infants are nose breathers and often have increased difficulty when they are congested. Nasal saline drops and gentle suctioning with a bulb syringe are often recommended. TEST-TAKING HINT: The test taker can eliminate answer 4 given a basic understanding of interventions to improve respiratory function.

7. The mother of a 5-year-old girl describes her daughter's symptoms to the nurse in the emergency room. She states that her daughter has had "a dry, hacking cough for the past 3 days that gets worse during the night." She further states that "now she is coughing up phlegm." Which discharge instruction does the nurse plan to provide? A. "Do not be surprised if she vomits her secretions." B. "Give your child cough drops as often as needed." C. "Return if she is not better after 3 days of antibiotics." D. "You can use a warm-mist humidifier in her room."

ANS: A This child has manifestations of bronchitis, which is frequently viral in nature. A dry, hacking cough gives way to a productive cough. Older children can be encouraged to cough up their secretions, but younger children often swallow them, leading to vomiting. Cough drops are not generally used, so as to encourage secretion mobilization. Because this disease is usually viral in nature, antibiotics are not routinely used. Cool-mist humidifiers are preferred over warm.

28. A nurse is assessing a 7-year-old who has white patches inside his mouth. Which question by the nurse would be most helpful to ask? A. "Do you have asthma?" B. "Do you drink milk?" C. "How much soda do you drink?" D. "When you do brush your teeth?"

ANS: A This child's complaint sounds like oral thrush. Often seen in infants, it can also be caused by inhaler use in children with asthma. The other questions are not related.

27. The clinic nurse is assessing a teenage girl who reports fever, chills, sore throat, and extreme fatigue during the last 2 weeks. Which focused assessment should the nurse perform? A. Assess lymph nodes. B. Collect buccal swabs. C. Obtain a urinalysis. D. Palpate the abdomen.

ANS: A This girl's age and symptoms are highly suggestive of infectious mononucleosis. The nurse should assess for swollen and tender occipital and cervical lymph nodes. The nurse should not palpate the abdomen because the spleen, if enlarged, can rupture under pressure. Buccal swabs and urinalysis are not related.

An adolescent is in the clinic with a few skin lesions on his thighs. The lesions have an open center with a thin crust. Which question by the nurse will aid the diagnosis of this patient's skin problem? A. "Are you active in any athletics or sports?" B. "Do you have siblings with respiratory infections?" C. "Do you seem to scratch a lot at night?" D. "Have you been in any wooded areas lately?"

ANS: A This lesion sounds like MRSA (methicillin-resistant Staphylococcus aureus). MRSA is common in athletes. This question would elicit the most useful information. Respiratory infection is not related. Scratching, especially at night, is characteristic of scabies. Wooded areas have ticks, which can spread Lyme disease.

26. A parent brings her child to the pediatric clinic and reports that the child has a rash on one side of his body that reminds her of chickenpox, but is more painful. Which medication does the nurse anticipate teaching the parent about? A. Acyclovir (Zovirax) B. Azathioprine (Imuran) C. Diphenhydramine (Benadryl) D. Intravenous immune globulin (IVIG)

ANS: A This rash sounds like herpes zoster (shingles), which is treated with acyclovir. Imuran is used in autoimmune disorders. Benadryl is used for itching. IVIG is also used in immune disorders.

2. A child is receiving vaccinations at a well-baby clinic. The nurse explains to the mother that the vaccinations provide which type of immunity? A. Active B. Innate C. Man-made D. Passive

ANS: A Vaccinations provide one type of active immunity. Passive immunity is brought about through immunoglobulins, either passed via the mother or given to the child through another means. Innate protection is provided by physical barriers, such as the skin or mucous membranes. Man-made immunity is not a classification of immunity.

The pediatric nurse is assessing a teenager's past medical history. Which information should the nurse inquire about? (Select all that apply.) A. All acute illnesses B. Any chronic illnesses C. Apgar scores at birth D. Birth weight E. Neonatal feeding problems

ANS: A, B Inquiring about all acute illness episodes and any chronic illnesses present is appropriate for a child of any age. Asking about Apgar scores, birth weight, and neonatal feeding problems would be important for younger children.

A nurse educator is discussing developmental theories with a student. What information is appropriate for the educator to provide regarding Lev Vygotsky's theory? (Select all that apply.) A. Children learn best with assistance from someone else. B. Culture plays a vital role in language development. C. Intelligence cannot be accurately measured. D. The ecological approach emphasizes three important systems. E. Motor skills are developed and refined in the preoperational stage.

ANS: A, B Vygotsky posited that learning takes place in a "zone of proximal development," which means that children learn best when assisted by another person. He also thought culture had a profound impact on language. Howard Gardner argued that intelligence cannot be measured by a single number from an IQ test. The ecological theory of Urie Bronfenbrenner emphasized three systems in a child's life (microsystem, mesosystem, exosystem). The preoperational stage is part of Jean Piaget's work.

The pediatric nurse is aware of the Core Strategies to reduce the spread of MRSA. Which actions do these strategies include? (Select all that apply.) A. Assess hand-hygiene practices. B. Implement contact precautions. C. Rapid reporting of MRSA laboratory results D. Recognize previously colonized patients. E. Screen all patients for MRSA.

ANS: A, B, C, D There are several core strategies to prevent the spread of MRSA, including assessing hand-hygiene practices, implementing contact precautions, rapid reporting of MRSA laboratory results, recognizing previously colonized patients, and educating health-care providers. Screening all patients is not one of the core strategies.

The nurse is preparing to provide information to the parents of a 14-year-old who is within normal limits for growth and development. What information is appropriate for the nurse to include? (Select all that apply.) A. Children of this age can anticipate long-term consequences of choices. B. Growth, although slowed, can still be significant. C. The child of this age may be able to give informed consent in some situations. D. The child of his age is not normally worried about sexual identity. E. Peer group influence is often stronger than family influence.

ANS: A, B, C, E Adolescents are in the stage of formal operations and can think abstractly. They are able to anticipate long-term consequences of choices. Growth continues during this period and, although slower than in other stages, can still be significant. Depending on state law and the teen's age, a teen may be able to give informed consent in situations such as seeking birth control or assistance with substance abuse. Establishing a sexual identity is one of the three main tasks of this age group. Peers often have more influence over the teen than family.

The parents of a school-age child with frequent sinus infections ask the nurse how to prevent the infections from occurring. Which questions by the nurse to the parents would elicit helpful information as to the cause of the problem? (Select all that apply.) A. "Do your other children have frequent infections?" B. "Has your child ever had allergy testing?" C. "Is your child around anyone who smokes?" D. "Was your child born prematurely or at term?" E. "Who watches your child before and after school?"

ANS: A, B, C, E Causes of sinusitis include allergies, exposure to other sick people (e.g., siblings or children at day care), exposure to cigarette smoke, and exposure to allergens. Eliminating these factors would help decrease the number of episodes of sinusitis, so it is appropriate to ask questions about these topics. Term versus premature birth is not a contributing factor.

A nurse is providing anticipatory guidance to a community parent group about burn prevention. When discussing school-age and older children, which cause of burn occurs most often in this age group? A. Cooking and kitchen activities B. Exposure to hot water C. Household electrical wires D. Touching hot appliances

ANS: A School-age and adolescent children are more independent than younger children and often cook food on the stove, in the oven, or in the microwave. This activity is associated with a risk for burns not seen in younger children. The other activities usually cause burns in infants and toddlers.

A mother is worried that her 3-month-old child is not holding her own head up. Which action by the nurse is most appropriate? A. Explain that sturdy head control occurs around 6 months. B. Document the findings and alert the health-care provider. C. Reassure the mother that her baby is completely normal. D. Teach the mother that head control is evident at 9 months.

ANS: A Sturdy head control occurs around 6 months of age. The findings should be documented, but there is no need to alert the health-care provider because this is a normal finding for a 3-month-old. Simply reassuring the mother does not give her the information she needs.

36. The parent of a 7-month-old baby who has been diagnosed with influenza asks the nurse if the baby can have Tamiflu (oseltamivir). Which response by the nurse is the most appropriate? A. "Children younger than 1 year old cannot take this medication," B. "I'm not sure; let's see how much your baby weighs today." C. "No, children can't take it because it contains aspirin products." D. "Yes, and you can take it, too, if you develop flu symptoms."

ANS: A Tamiflu is contraindicated in children younger than 1 year old. The nurse should advise the parent of this fact.

A nurse is assessing a school-age child in the clinic with an earache and fever. Using the SODA mnemonic, what question by the nurse best relates to "S"? A. "Does it keep you from sleeping?" B. "Has this affected your schoolwork?" C. "How long have you been sick?" D. "How sore is your ear today?"

ANS: A The SODA mnemonic stands for sleep, output, diet, and activity. Asking if the child's earache is affecting sleep is the appropriate question for the "S" component of this mnemonic.

A home health-care nurse sees several pediatric patients who have the nursing diagnosis of delayed growth and development. Which action by a child would indicate that outcomes have been met? A. A 3-year-old child walks backward. B. A 4-year-old child plays video games. C. A 5-year-old child can unscrew items. D. A 7-year-old child uses scissors to cut an outline figure.

ANS: A The ability to walk backward is accomplished in the toddler stage (1-3 years). The ability to play video games is more likely seen in the adolescent stage. Screwing and unscrewing objects is appropriate for a toddler. Using scissors to accurately cut a figure from an outline is a milestone of the early childhood stage (3-6 years).

35. A father calls the pediatric clinic to report that his child was diagnosed with influenza at an urgent care facility yesterday and was prescribed oseltamivir (Tamiflu). The father worries that the dose of 75 mg twice a day is too high. He reports that his child weighs 90 lb (40.9 kg). Which response by the nurse is best? A. "No, that is an appropriate dose for your child." B. "No, that is way too little medicine for your child." C. "Tamiflu should not be used in children under 100 lb." D. "Yes, that is double the normal dose for a child."

ANS: A The dose of Tamiflu for children weighing over 88 lb (40 kg) is 75 mg twice a day. The nurse should advise the father that the dose is appropriate.

24. The pediatric nurse is discharging a child diagnosed with cytomegalovirus infection (CMV). Which teaching is most appropriate for this child? A. Ensure adequate rest. B. Keep the child isolated. C. Offer favorite foods. D. Provide plenty of fluids.

ANS: A The most common problem for children during the convalescent phase after acute CMV infection is fatigue. The nurse teaches the parents to ensure the child gets plenty of rest. Adequate nutrition and hydration are always important and are not specific for this condition. The child does not need to be isolated.

39. A child has been hospitalized with rubella. Which action by the charge nurse is most appropriate? A. Do not allow pregnant nursing staff in the room. B. Inform the parents that fresh produce is not allowed. C. Place the child on contact isolation precautions. D. Use standard precautions when caring for the child.

ANS: A The most serious consequence of rubella infection occurs prenatally; exposure in utero can lead to cognitive impairment, deafness, eye disorders, cardiac defects, and stillbirth. Pregnant staff should not enter this room. Disallowing produce is not related to this disorder. Contact precautions are not warranted; this disease is spread through the airborne route. Standard precautions are used for all patients.

A teen has been taking griseofulvin (Fulvicin) for a fungal skin and scalp condition and reports that it is not working. Which action by the nurse is the most important? A. Assess for noncompliance with the regimen. B. Discuss changing medications with the provider. C. Prepare to administer the drug as an injection. D. Tell the teen it takes nearly 6 months to work.

ANS: A The nurse should assess the teen for compliance with the medication regimen, because it takes at least 6 weeks for this medication to work. Due to the lengthy treatment time and the fact that teens are often noncompliant, this would be the nurse's first step. If the medication has been used appropriately, discussing a change would be correct. This drug is not give via IM injection.

1. Which statement by the nursing student indicates a correct understanding of the anatomy and physiology of the respiratory system? A. Air passes through the trachea into the bronchus. B. Oxygen exchange with the bloodstream occurs in the veins. C. Oxygen passes from the larynx to the pharynx. D. The bronchus divides into smaller branches or acini.

ANS: A The oxygen passes from the pharynx to the larynx. From the larynx, air passes through the trachea, which branches into the left and right bronchi. Each bronchus divides into smaller branches called bronchioles. The bronchioles end in a cluster of air sacs called acini. Oxygen exchange with the bloodstream occurs in the capillaries.

A pediatric nurse needs to administer acetaminophen (Children's Tylenol) to patients in the intensive care unit (ICU). Which dose, based on age, is correct? A. 0 to 3 months, 40 mg B. 4 to 11 months, 220 mg C. 2 to 3 years, 120 mg D. 4 to 5 years, 100 mg

ANS: A The proper dosage based on age is 0 to 3 months, 40 mg; 4 to 11 months, 80 mg; 12 to 23 months, 120 mg; 2 to 3 years, 160 mg; and 4 to 5 years, 240 mg.

46. The pediatric nurse is observing a student nurse teach a child how to use a peak flow meter. Which instruction by the student requires intervention by the pediatric nurse? A. "Exhale for as long as you can to empty your lungs." B. "Keep your tongue away from the mouthpiece." C. "Stand up straight and tall when using the meter." D. "Write down the highest of the three readings."

ANS: A The proper way to use the peak flow meter is to blow hard and fast when exhaling. The nurse should intervene when the student states otherwise. The other instructions are correct.

A child will be hospitalized in the following week. In order to decrease the child's and parents' stress related to the hospitalization, which action by the clinic nurse would be most helpful? A. Arrange for the family to visit the hospital and have a tour. B. Give the family written information on visiting hours. C. Introduce the family to another family whose child is hospitalized. D. Suggest the family "take a break" and not stay with the child.

ANS: A There are several ways to decrease the stress of a planned hospitalization, one of which is to arrange a visit to the hospital where all involved can have a tour and visit the unit where the child will be staying. Written information is always good, but is not the best way to diminish stress. Some families may be open to being introduced to others, but the nurse must take care to obtain permission first or risk a HIPAA violation. Parents are the main source of comfort for their children, so often a parent will stay with a hospitalized child.

A nurse is teaching parents about appropriate discipline for their toddler. Which information is appropriate for the nurse to include in the session? (Select all that apply.) A. Be firm and specific but respectful. B. Deliver consequences immediately. C. Tie consequences to the action if possible. D. Time-outs are 5 minutes for each year of life. E. Try to anticipate and avoid tantrums.

ANS: A, B, C, E Effective discipline involves parents being firm and specific. They should be respectful and speak as they would want to be spoken to. The most effective consequences are delivered immediately and are tied to the action in a logical way. If parents can recognize triggers for temper tantrums, they can avoid them (e.g., a child who is tired needs a nap or bedtime). A rule of thumb for time-out is 1 minute for each year of life.

An 8-year-old child is in the clinic and is diagnosed with ringworm. Which medications does the nurse anticipate teaching the child and parents about? (Select all that apply.) A. Griseofulvin (Fulvicin) B. Infliximab (Remicade) C. Ketaconazole (Selenium) D. Naproxen (Naprosyn) E. Salicylates (Aspirin)

ANS: A, C A child diagnosed with ringworm will require teaching regarding antifungal medications. Griseofulvin (Fulvicin) and ketaconazole (Selenium) are appropriate medications to include in the teaching session. The other medications are not used to treat ringworm.

Which child diagnosed with pneumonia would benefit most from hospitalization? 1. 13-year-old who is coughing, has coarse breath sounds, and is not sleeping well 2. 14-year-old with a fever of 38.6°C (101.5°F), rapid breathing, and a decreased appetite. 3. 15-year-old who has been vomiting for 3 days and has a fever of 38.5°C (101.3°F). 4. A 16-year-old who has a cough, chills, fever of 38.5°C (101.3°F), and wheezing.

3. The teen who has been vomiting for several days and is unable to tolerate oral fluids and medication should be admitted for intravenous hydration. TEST-TAKING HINT: The test taker can eliminate answers 1, 2, and 4 if familiar with the common signs and symptoms of pneumonia.

A parent asks how to care for a child at home who has the diagnosis of viral tonsillitis. Which is the nurse's best response? 1. "You will need to give your child a prescribed antibiotic for 10 days." 2. "You will need to schedule a follow-up appointment in 2 weeks." 3. "You can give your child Tylenol every 4 to 6 hours as needed for pain." 4. "You can place warm towels around your child's neck for comfort."

3. Tylenol is recommended PRN for pain relief. TEST-TAKING HINT: The test taker can eliminate answer 1 by knowing that antibiotics are not given for viral illnesses. Answer 4 can be eliminated by knowing that swelling and inflammation increase with heat. Cold causes vasoconstriction of the vessels, aiding in decreasing the amount of inflammation.

Which is the nurse's best response to the parent of an infant diagnosed with the first otitis media who wonders about long-term effects? 1. "The child could suffer hearing loss." 2. "The child could suffer some speech delays." 3. "The child could suffer recurrent ear infections." 4. "The child could require ear tubes."

3. When children acquire an ear infection at such a young age, there is an increased risk of recurrent infections. TEST-TAKING HINT: Answers 1, 2, and 4 can be eliminated if the test taker understands that these are all long-term effects of recurrent ear infections. The question is asking about a single incident of otitis.

32. A child has been diagnosed with influenza and is prescribed oseltamivir (Tamiflu). Which instruction by the nurse is most important? A. "Do not use aspirin with this drug." B. "Encourage plenty of liquids." C. "Rinse the inhaler after each use." D. "This will cure the flu in 5 days."

ANS: B Common side effects of Tamiflu include nausea, vomiting, GI distress, and diarrhea. The child should drink plenty of fluids to avoid dehydration. Aspirin is not used in children at all due to the risk of Reye's syndrome. Tamiflu is not given via inhaler. The medication is not curative.

26. A child has been seen in the clinic and diagnosed with bronchitis. The next day, the mother calls the clinic to ask about using cough medicine to decrease the child's cough. Which response by the nurse is the most appropriate? A. "Children should never use cough medicine for bronchitis." B. "If she can't sleep due to a cough, a dose would be okay at night." C. "It would be alright to use, but watch her for hyperactivity." D. "Yes, give her over-the-counter cough medicine every 4 hours."

ANS: B Cough medication is used with caution in children with bronchitis in order to facilitate secretion removal. However, if the child cannot sleep due to coughing, a dose at night would be appropriate. Many cough medications contain alcohol, which could lead to sleepiness.

16. A nurse is teaching the parents of a 10-year-old child diagnosed with cystic fibrosis. Which instruction by the nurse is most appropriate? A. "For Pseudomonas infections, we can use penicillin antibiotics." B. "Preventing respiratory infections is crucial for quality of life." C. "Unfortunately, your child is sterile and unable to have children." D. "With pancreatic enzymes, vitamin replacement is not needed."

ANS: B Cystic fibrosis (CF) is characterized by frequent, severe respiratory infections, often caused by Pseudomonas, which is treated with tobramycin (TOBI) or azithromycin (Zithromax). Preventing respiratory infections is a crucial part of caring for the child with CF. Reproduction is often affected in people with CF, but without testing, it is impossible to say that the child is sterile. Vitamin replacement is needed along with pancreatic enzyme replacement.

A child has been diagnosed with scabies and the parents are taught about the use of 5% permethrin lotion (Elimite). Which statement by the parents indicates the need for further instruction? A. "Elimite goes from the nape of the neck to the toes, except on the genitals." B. "The lotion is left on until the next day, when a bath is given." C. "This treatment will have to be repeated 1 week after the first." D. "We will give our child a warm soapy bath before applying the lotion."

ANS: B Elimite is left on for 8 to 14 hours and then must be removed. The other statements show good understanding.

23. An immunocompromised child has been admitted to the hospital with Fifth's disease. Which action by the nurse is most appropriate? A. Place the child in contact precautions. B. Place the child in droplet precautions. C. Place the child in protective isolation. D. Place the child on standard precautions.

ANS: B Fifth's disease is spread through respiratory droplets, so droplet precautions are appropriate. Of course standard precautions should be used with all patients, but this is not enough in this situation. Contact and protective precautions are not needed for this disease.

21. A 7-year-old child is brought to the clinic with what the parent describes as "tonsillitis." The child has a moderate fever, foul breath, and dysphagia, and occasionally spits up lumps of foul-smelling material. Which medication does the nurse prepare to instruct the parents on based on the child's symptoms? A. Amoxicillin (Amoxil) B. Clindamycin (Cleocin) C. Erythromycin (Erythrocin) D. Tetracycline (Sumycin)

ANS: B For acute tonsillitis, penicillin, amoxicillin, or erythromycin are the most commonly prescribed antibiotics. However, with the history of this child spitting up lumps of foul-smelling material, it is more likely he or she has chronic tonsillitis, which is best treated with cephalosporin or clindamycin.

29. An infant is being discharged from the hospital after treatment for respiratory syncytial virus (RSV). The infant still has some mild respiratory distress at times. Which discharge instruction is the priority for this infant? A. "Bring the child back if she runs a temperature." B. "Feed baby small amounts while she is sitting up." C. "Give her antibiotics right after feeding her." D. "If you need to use the bulb suction, bring her back."

ANS: B Guidelines for feeding a child with respiratory distress revolve around preventing aspiration. The child should be fed in an upright position and given small amounts, perhaps more often. An elevated temperature is usually seen in RSV. Antibiotics are not generally used for this disease. Parents needs to know how to use the bulb suction to help manage the accompanying rhinitis.

14. An 8-day-old was admitted to the hospital with vomiting and dehydration. The newborn's heart rate is 170, respiratory rate is 44, blood pressure is 85/52, and temperature is 99°F (37.2°C). What is the nurse's best response to the parents who ask if the vital signs are normal? 1. "The blood pressure is elevated, but the other vital signs are within normal limits." 2. "The temperature is elevated, but the other vital signs are within normal limits." 3. "The respiratory rate is elevated, but the other vital signs are within normal limits." 4. "The heart rate is elevated, but the other vital signs are within normal limits."

4. "The heart rate is elevated, but the other vital signs are within normal limits."

37. A 3-year-old child is brought to the emergency department with sudden onset of hoarseness, wheezing, and cough. The child has a history of asthma, but the parent is worried about an aspirated toy part. Which action by the nurse would quickly differentiate between the two? A. Administer an inhaled bronchodilator. B. Auscultate lungs for unilateral wheezing. C. Obtain a stat portable chest x-ray. D. Quickly obtain an oxygen saturation.

ANS: B In foreign body aspiration (FBA), wheezing would more likely be heard unilaterally. In an asthma exacerbation, wheezing would be bilateral. The other measures are appropriate care measures in either case, but the quickest way to assess for FBA is to auscultate the lungs.

42. An 11-year-old child who has a history of asthma is brought to the family practice clinic by a parent who reports that the child has white plaque over the inside of the mouth and coating the tongue. Which question by the nurse would elicit the most useful information? A. "Are you brushing your teeth and tongue regularly?" B. "Do you rinse your mouth after using your inhaler?" C. "Have you ever had something like this before?" D. "When was the last time you went to the dentist?"

ANS: B Use of inhalers for asthma, especially corticosteroids, can cause yeast infection (candidiasis) if the patient does not rinse the mouth afterward. The other questions are not helpful in this situation.

A child has been hospitalized with rubeola. Which actions by the nursing staff are most important? (Select all that apply.) A. Administer ordered antibiotics on time. B. Assess the child for Koplik's spots. C. Ensure the room is dark for photophobia. D. Monitor the child for febrile seizures. E. Report the disease to health authorities.

ANS: B, C, E Appropriate nursing care for the child with rubeola includes assessing the child's mouth for Koplik's spots, providing comfort for photophobia by darkening the room, and reporting the disease to authorities. Rubeola is a viral disease not treated with antibiotics. Fever is moderate and seizures are not usually seen.

The pediatric nurse is examining a newborn infant and notes a turning in of the foot and turning out of the toes when the sole of the foot is stroked. Which action by the nurse is most appropriate? A. Arrange a consultation with a developmental specialist. B. Assess the parents' family histories for genetic defects. C. Document the findings in the patient's chart. D. Instruct the parents on required follow-up care.

ANS: C The newborn is exhibiting the Babinski reflex, one of the normal primitive reflexes that should disappear by 9 months of age. Documentation is all that is required.

The parents of a 6-year-old who has a new diagnosis of asthma ask the nurse what to do to make their home a more allergy-free environment. Which is the nurse's best response? 1. "Use a humidifier in your child's room." 2. "Have your carpet cleaned chemically once a month." 3. "Wash household pets weekly." 4. "Avoid purchasing upholstered furniture."

4. Leather furniture is recommended rather than upholstered furniture. Upholstered furniture can harbor large amounts of dust, whereas leather furniture may be wiped off regularly with a damp cloth. TEST-TAKING HINT: The test taker can eliminate answer 3 because there is no known way to make a pet allergy-free. Household pets are discouraged for all children with asthma or severe allergies. Answer 2 can be eliminated if the test taker understands that chemical agents are triggers to asthma for many children.

A child is complaining of throat pain. Which statement by the mother indicates that she needs more education regarding the care and treatment of her daughter's pharyngitis? 1. "I will have my daughter gargle with salt water three times a day." 2. "I will offer my daughter ice chips several times a day." 3. "I will give my daughter Tylenol every 4 to 6 hours as needed." 4. "I will ask the nurse practitioner for some amoxicillin."

4. Pharyngitis is a self-limiting viral illness that does not require antibiotic therapy. Pharyngitis should be treated with rest and comfort measures, including Tylenol, throat sprays, cold liquids, and Popsicles. TEST-TAKING HINT: Answers 1, 2, and 3 are comfort measures. The question requires that the student have knowledge regarding pharyngitis.

30. Which approach should the nurse use to gather information from a child brought to the ED for suspected child abuse? 1. Promise the child that her parents will not know what she tells the nurse. 2. Promise the child that she will not have to see the suspected abuser again. 3. Use correct anatomical terms to discuss body parts. 4. Tell the child that the abuse is not her fault and that she is a good person.

4. Tell the child that the abuse is not her fault and that she is a good person.

24. According to developmental theories, which important event is essential to the development of the toddler? 1. The child learns to feed self. 2. The child develops friendships. 3. The child learns to walk. 4. The child participates in being potty-trained.

4. The child participates in being potty-trained. Developmental theorists like Erickson and Freud believe that toilet training is the essential event that must be mastered by the toddler.

What is the most important piece of information that the nurse must ask the parent of a child in status asthmaticus? 1. "What time did your child eat last?" 2. "Has your child been exposed to any of the usual asthma triggers?" 3. "When was your child last admitted to the hospital for asthma?" 4. "When was your child's last dose of medication?"

4. The nurse needs to know what medication the child had last and when the child took it in order to know how to begin treatment for the current asthmatic condition. TEST-TAKING HINT: Whereas all of the information here is essential, answer 4 gives the most important information. The test taker can eliminate answers 2 and 3 because the responses to these inquiries have no direct impact on the immediate treatment of the child. These two answers give information about the severity of the child's illness, but they do not affect the immediate treatment plan. Answer 4 is essential to deciding what medication should be given the child to relieve the current symptoms.

An infant is not sleeping well, crying frequently, has yellow drainage from the ear, and is diagnosed with an ear infection. Which nursing objective is the priority for the family? 1. Educating the parents about signs and symptoms of an ear infection. 2. Providing emotional support for the parents. 3. Providing pain relief for the child. 4. Promoting the flow of drainage from the ear.

3. Providing pain relief for the infant is essential. With pain relief, the child will likely stop crying and rest better. TEST-TAKING HINT: The test taker needs to consider the needs of the child and the parent at this time. If the pain is controlled, the parents and child will both be in a better state. The other items are all essential in providing care for the child with otitis, but pain relief offers the best opportunity for the child and the parent to return to normal conditions

How will a child with respiratory distress and stridor and who is diagnosed with RSV be treated? 1. Intravenous antibiotics. 2. Intravenous steroids. 3. Nebulized racemic epinephrine. 4. Alternating doses of Tylenol and Motrin.

3. Racemic epinephrine promotes mucosal vasoconstriction. TEST-TAKING HINT: This is a knowledgelevel question that requires the test taker to know how RSV is treated.

5. Which statement accurately describes the best method for assessing a 12-month-old? 1. The nurse should assess the child on the examining table. 2. The nurse should assess the child in a head-to-toe sequence. 3. The nurse should have the child's mother assist in holding her down. 4. The nurse should assess the child while she is in her mother's lap.

4. The nurse should assess the child while she is in her mother's lap. Infants are most secure when in prox- imity to the parent. The parent's lap is an excellent place to assess the child.

Which statement by the parents of a toddler with repeated otitis media indicates they need additional teaching? 1. "If I quit smoking, my child may have a decreased chance of getting an ear infection." 2. "As my child gets older, he should have fewer ear infections, because his immune system will be more developed." 3. "My child will have fewer ear infections if he has his tonsils removed." 4. "My child may need a speech evaluation.

3. Removing children's tonsils may not have any effect on their ear infection. Children who have repeated bouts of tonsillitis can have ear infections secondary to the tonsillitis, but there is no indication in this question that the child has a problem with tonsillitis. TEST-TAKING HINT: The test taker can eliminate answers 1, 2, and 4 because those options are true.

A parent asks the nurse how it will be determined if their child has respiratory syncytial virus (RSV). Which is the nurse's best response? 1. "We will do a simple blood test to determine whether your child has RSV." 2. "There is no specific test for RSV. The diagnosis is made based on the child's symptoms." 3. "We will swab your child's nose and send that specimen for testing." 4. "We will have to send a viral culture to an outside lab for testing."

3. The child is swabbed for nasal secretions. The secretions are tested to determine if a child has RSV. TEST-TAKING HINT: The test taker can eliminate answers 1, 2, and 4 because the child's nasal sections will be swabbed.

A child's parent asks the nurse what treatment the child will need for the diagnosis of strep throat. Which is the nurse's best response? 1. "Your child will be sent home on bedrest and should recover in a few days without any intervention." 2. "Your child will need to have the tonsils removed to prevent future strep infections." 3. "Your child will need oral penicillin for 10 days and should feel better in a few days." 4. "Your child will need to be admitted to the hospital for 5 days of intravenous antibiotics."

3. The child will need a 10-day course of penicillin to treat the strep infection. It is essential that the nurse always tell the family that, although the child will feel better in a few days, the entire course of antibiotics must be completed. TEST-TAKING HINT: Answer 2 can be eliminated because it is a treatment for recurrent tonsillitis, not strep throat. Answer 1 can be eliminated if the test taker understands that bacterial infections need to be treated with antibiotics.

22. A child had a tonsillectomy this morning. What action by the nurse is most important for the child's safety? A. Avoid giving her red popsicles. B. Limit activity the first night. C. Offer ice cream when awake. D. Position the child on her side.

ANS: D After tonsillectomy, children are placed on their side to facilitate drainage and prevent aspiration. Maintaining a patent airway is the priority. Red-colored foods or fluids are not given, as the nurse may not be able to differentiate between the food and bloody drainage; however, this is not a priority for safety. Activity should be limited, but this does not take priority over maintaining the airway. Ice cream and other dairy products are not given because they coat the throat and usually cause coughing or throat clearing, which can lead to bleeding.

A child needs hearing assessments. To assess air and bone conduction of sound, which assessment technique is most appropriate? A. Have the child place a block into a box each time he or she hears a sound. B. Place a probe into the ear canal and measure the amount of sound reflected. C. Strike a tuning fork and place the handle against the back of the child's head. D. Strike a tuning fork, place it on the mastoid process, then move it to within1 inch of the ear canal.

ANS: D Air and bone conduction of sound are assessed with both the Weber test and the Rinne test. The Rinne test uses as vibrating tuning fork placed against the child's mastoid process. When the child can no longer hear the fork, the nurse moves it to within 1-2 inches of the auditory meatus. The child should hear this sound twice as long as the bone-conducted sound. The Weber test uses the vibrating tuning fork placed against the midline on top of the child's head. Engaging in a play activity related to hearing sounds is called conditioned-play audiometry and assesses hearing. Using a probe in the ear canal is called tympanometry and assesses the status of the middle ear.

4. A nurse is providing anticipatory guidance to new parents. Which instruction by the nurse will assist the parents in maintaining physical barriers to prevent infection in their newborn? A. Breastfeeding provides some antibodies. B. Ensure your baby is getting enough nutrition. C. Keep your baby away from people who are sick. D. Wash your baby with gentle soap and dry well.

ANS: D All options are sound advice for helping to keep a newborn well. However, the only option specific to physical barriers available to protect against infection (skin, mucous membranes) is to wash the baby's skin with gentle soap and dry it well, helping to keep it intact.

A nurse is working with a child who has a chronic skin disorder consisting of many vesicles and pustules. Which nursing assessment indicates that a priority long-term goal has been met? A. Child states that he no longer gets teased at school because of his appearance. B. Parents and child verbalize acceptance of disease process and need for medication. C. Patient participates in sporting events and other after school-activities regularly. D. Skin around primary lesions remains free of redness, warmth, swelling, and pain.

ANS: D All options show evidence of positive outcomes. However, physical needs take priority over psychosocial needs, so absence of infection is evidence that a priority goal has been met. Secondary infection can occur due to scratching, picking, and the presence of the lesions themselves.

4. A school-age child has asthma and lives in a home where both parents smoke. The nurse has provided extensive education to the parents on the dangers of second-hand smoke. Which assessment by the nurse indicates that goals for a family nursing diagnosis have been met? A. Child's clothing no longer smells of cigarette smoke B. Father states he has quit smoking; mother has cut down C. Parents state they smoke only in the basement now D. Significant decrease in asthma "attacks" over a year

ANS: D An important goal for this family is understanding how to avoid exposing their child to second-hand smoke. The only objective assessment data to show that the child is not exposed to smoke is the decrease in asthma "attacks" the child has had in the last year. New clothing will not smell like smoke; if the mother continues to smoke even in lesser amounts, the child will still be exposed; and smoking anywhere in the house pollutes all the air in the house.

A child has had eye testing. The nurse reads in the child's chart that the Hirschberg test demonstrated displacement of light reflection in one eye. What does this indicate to the nurse? A. Color blindness B. Normal ocular alignment C. Presence of cataracts D. Presence of strabismus

ANS: D Ocular alignment is demonstrated through the Hirschberg test. When a light is shone directly into the child's eyes, the reflection should fall in the same location on the cornea of both eyes. Displacement of the corneal light reflection is indicative of strabismus. Color blindness is assessed with the Ishihara pseudochromatic charts. Cataracts are assessed via the red reflex.

34. A parent is refusing to have a child vaccinated, preferring to have the child contract the illness and develop "natural immunity." Which response by the nurse is best? A. "I'm sure you know what is best for your baby." B. "I'll have to report you to social work." C. "That practice is dangerous and illegal." D. "These diseases have many serious consequences."

ANS: D Parents do have the right to refuse vaccinations, but the nurse has the responsibility of ensuring the parents have adequate information about the diseases and vaccinations. Informing the parent about possible consequences of contracting a disease is an important part of this job. The nurse should not just acquiesce and say the parent knows best without educating him or her. The other two options are threatening, and it is not illegal for a parent to opt out of vaccinations.

15. A father brings his 1-year-old son to the clinic and states that when he kisses the child's cheek, it tastes salty. Which diagnostic test does the nurse educate the father on based on the father's statement? A. Large bowel barium series B. Pancreatic enzyme analysis C. Pulmonary function studies D. Quantitative sweat chloride test

ANS: D Salty-tasting sweat and tears are a characteristic finding in cystic fibrosis. The diagnostic test for this disorder is the quantitative sweat chloride test. Pancreatic enzyme studies are invasive and not usually performed on children. Pulmonary function studies are done in older children who can cooperate. Large bowel barium studies are not needed.

A child presents to the emergency department with cutaneous manifestations of an allergic response. Which nursing diagnosis takes priority? A. Altered comfort related to itching lesions B. Impaired tissue integrity related to lesions C. Knowledge deficit related to unknown condition D. Risk for impaired airway secondary to edema

ANS: D Usually, actual nursing diagnoses take priority over "risk for" diagnoses, but in this case, risk for impaired airway is the priority. The child's airway can quickly become compromised due to laryngeal edema. The nurse places priority on frequent assessments of the child's airway. The other diagnoses are relevant, but not as important as the potential for compromised airway.

A nurse is attempting to assess a toddler, who is being uncooperative. What action by the nurse would be best to accomplish this task? A. Get on the floor while assessing the child. B. Give the child toys to play with. C. Have the parent restrain the toddler. D. Visit with the parent for a short while.

ANS: D Young children need to feel comfortable with the nurse before they will be cooperative. At this age, the best way to improve the child's comfort level is for the nurse to establish a rapport with the parent(s). Once the child becomes comfortable with the nurse present, he or she is more likely to cooperate. Giving toys and getting on the same level of the child are helpful, age-appropriate actions, but not the best answer. Having the parent restrain the child would be the last resort unless the assessment technique could injure a struggling child (e.g., otoscopic examination of the ear).

A nurse is assessing several children during a shift at the well-child clinic. Which child demonstrates successful resolution of the Erikson stage of autonomy versus shame and doubt? A. A 15-month-old playing on the floor with supervision B. An 18-month-old being consistently consoled by her father C. A 20-month-old using building blocks with her grandfather D. A 24-month-old being allowed to independently dress himself

ANS: D The Erikson stage of autonomy versus shame and doubt occurs during the ages of 1 to 3. During this stage, the child develops a "can-do" attitude and wishes to be independent. A child who does not successfully meet the tasks of this stage will suffer from self-doubt later on. The most independent child in the options is the 24-month-old dressing independently. This child is mastering the tasks of this developmental stage.

A nurse has been asked to perform a Romberg test on a school-age child. What action does the nurse take to perform this assessment? A. Ask the child to smile, frown, and make other faces. B. Have the child touch a finger to the nose several times. C. Instruct the child to walk across the room and back. D. Tell the child to stand, close his or her eyes, and hold the arms out in front.

ANS: D The Romberg test assesses cerebellar functioning and is performed by asking the child to stand, shut the eyes, and then hold the arms outstretched in front. A normal result is performing these actions without swaying. Looking at facial expressions tests cranial nerve VII. The finger-to-nose test also assesses cerebellar function. Observing posture and gain gives information about the child's musculoskeletal system.

A nurse is assessing a 10-month-old baby's anterior fontanel and finds it slightly depressed; the fontanel measures 2 inches (5.08 cm). What conclusion and action are most appropriate? A. Delayed closing; alert health-care provider. B. Fontanel is closing; document findings. C. Large for age; assess for Down's syndrome. D. Sign of dehydration; assess fluid status.

ANS: D The anterior fontanel remains open until 12-18 months of age. The normal size is 0.4-2.8 inches (1-7 cm). A depressed fontanel is a sign of possible dehydration, and the nurse should assess for other signs of fluid status. The closing is not delayed, it is not overly large, nor should it be closing as part of normal growth and development.

24. An emergency department nurse is supervising a nursing student who is preparing a racemic epinephrine (MicroNefrin) nebulizer treatment for a child with stridor from suspected epiglottitis. The student prepares 0.75 mg in 2 mL of normal saline. Which action by the registered nurse is most appropriate? A. Allow the student to administer the nebulizer treatment. B. Have the student explain the action and side effects of the drug. C. Remind the student to assess lung sounds after the treatment. D. Tell the student to look up the drug dosage information again.

ANS: D The correct dose of racemic epinephrine is 0.25-0.5 mg in 3 mL of normal saline. The nurse should have the student look up the dosing information again. The other actions would be appropriate if the student had the correct dose.

7. A child with a primary immunodeficiency disorder had postimmunization titers drawn. The titers came back low. Which explanation does the nurse give the parents? A. "The immunizations had no effect on the child." B. "This result indicates a hyperactive response." C. "Vaccinations are not needed in your child." D. "Your child's immune system did not respond."

ANS: D The most correct answer is that the low titers indicate that the child's immune system did not respond adequately to the vaccinations. It cannot be determined if they had no effect at all on the child. Although vaccinations did not produce the desired response, that does not mean they are not needed; they just did not work as planned. This child had a hypoactive, not hyperactive, response.

35. A nurse has given an infant a vaccination. Which information is important to document specifically for this vaccination? A. Date of next regularly scheduled immunization B. Drug, dose, site of administration, infant's reaction C. Parental education provided before administration D. Vaccine information sheet given before administration

ANS: D The nurse is legally required to provide the appropriate vaccine information sheet to the parent/guardian prior to administering a vaccination. The other information is important to document too, but is not specific for vaccinations

A preschool-age child is going to have a potentially painful procedure. What action by the nurse is best to prepare the child for this event? A. Allow the child to decide if the parents stay or not. B. Let the child touch and explore the equipment first. C. Talk about it briefly for several days beforehand. D. Use play to demonstrate the procedure to the child.

ANS: D The nurse should use play to demonstrate the procedure and allow the child to "perform" the procedure on a doll or stuffed toy. The parents should be given the choice of staying in the room or not. Preparation should begin immediately before the procedure to keep the child from worrying about it for hours or days. An older child should be allowed to look at and explore equipment that is going to be used.

A mother and 1-year-old child are being seen in the well-child clinic. The mother asks the nurse for guidance in setting an appropriate bedtime for the child. She and her husband have tried several different times and can't seem to agree on the right time. Which response by the nurse is the most appropriate? A. "As long as it's consistent, it doesn't matter when bedtime is." B. "At this age, your child needs 10 to 12 hours of sleep each night." C. "Children get sleepy when warm, so dress your child heavily for bed." D. "What 'sleepy' behaviors does your child show when tired?"

ANS: D The parents should note any signs of fatigue in a child, such as rubbing eyes, temper tantrums, yawning, or fussiness. They should use this information to help set a routine bedtime. At this age, children need between 12 and 16 hours of sleep a night. Consistency is important, but the bedtime needs to be early enough to ensure adequate sleep. Children, especially infants, should not be over-warm when sleeping.

A toddler has a few vesicular lesions on his hands that rupture, producing a honey-colored, sticky exudate. Based on this assessment, which medication does the nurse teach the parents about? A. Azithromycin (Zithromax) B. Amoxicillin/clavulanate (Augmentin) C. Doxycycline (Vibramycin) D. Mupirocin (Bactroban) ointment

ANS: D This child has impetigo, which is treated in one of two ways. For limited lesions (which this child has), topical Bactroban is appropriate. For widespread lesions, Augmentin is one of the antibiotics used. Azithromycin and doxycycline are not used

7. A nurse is working with the parents of a child who has the following complaints: fatigue, insomnia, persistent irritable mood, and significant weight loss. Which discharge instruction is the priority? A. Encourage the child to eat more protein. B. Enroll the child in after-school activities. C. Establish a routine bedtime and wake time. D. Ensure the child is safe at all times.

ANS: D This child has symptoms of depression. The priority is ensuring the child's safety.

A child who was visiting a wooded area on vacation presents to the doctor's office with a localized bull's-eye rash. Which action by the nurse is the most appropriate? A. Advise parents that treatment depends on laboratory results. B. Assess for cardiac and neurological involvement. C. Facilitate admission to a nearby hospital. D. Teach the parents about cefuroxime (Ceftin).

ANS: D This child may have early Lyme disease, which is treated presumptively with Ceftin prior to obtaining laboratory results. By the time carditis and neurological involvement occur, the rash has generalized. Admission is not warranted.

27. Four children are in the pediatric clinic waiting to be seen. Which patient should the nurse see first? A. Afebrile, parent reports harsh barky cough B. Pulling on ear, temperature 103°F (39.5°C) C. Salty-tasting sweat, poor weight gain D. Wheezing, retracting, no wet diapers today

ANS: D This child not only has a respiratory problem, he or she is also dehydrated and is the sickest of the four. The nurse should see this patient first. The child with the barky cough is afebrile and has no signs of respiratory distress. Pulling on the ear indicates an ear infection, and these children often run high fevers. The child with salty-tasting sweat and poor weight gain is demonstrating a chronic condition, most likely cystic fibrosis.

29. A child is hospitalized with a serious bacterial infection. Which assessment finding indicates that the goals for a priority nursing diagnosis have been met? A. Intact skin integrity B. Normal temperature C. Stable weight D. Urine output of 1 mL/kg/hour

ANS: D This urine output is normal, demonstrating that the goals for the diagnosis of risk for fluid volume deficit have been met. The other outcomes are demonstrative of met goals, but do not take priority over a possible fluid volume deficit.

A 5-year-old child is having an acute asthma attack. How does the nurse position the child while waiting for a respiratory treatment? A. Prone across the parent's lap B. Semi-Fowler's position in bed C. Upright in a hard-backed chair D. Upright in the tripod position

ANS: D Tripod positioning is often seen in children with respiratory distress. In this position the child sits upright leaning forward on outstretched arms with the jaw thrust forward. This position maximizes airway opening and use of accessory muscles. The nurse can assist the child into this position. The other positions will not be as helpful. However, it is important to note that because children having respiratory distress are often anxious, it is important to allow the child to assume the position in which he or she is most comfortable.

What should be the nurse's first action with a child who has a high fever, dysphagia, drooling, tachycardia, and tachypnea? 1. Immediate IV placement. 2. Immediate respiratory treatment. 3. Thorough physical assessment. 4. Lateral neck radiographs..

4. This child is exhibiting signs and symptoms of epiglottitis and should be kept as comfortable as possible. The child should be allowed to remain in the parent's lap until a lateral neck film is obtained for a definitive diagnosis TEST-TAKING HINT: The test taker must accurately identify that the question is describing a child with epiglottitis and understand that agitation in this child can result in complete airway obstruction.

Which intervention is most appropriate to teach the mother of a child diagnosed with a URI and a dry hacking cough that prevents him from sleeping? 1. Give cough suppressants at night. 2. Give an expectorant every 4 hours. 3. Give cold and flu medication every 8 hours. 4. Give 1/2 teaspoon of honey four to five times per day

4. Warm fluids, humidification, and honey are best treatments for a URI. TEST-TAKING HINT: The latest recommendations for treatment of URIs in children are to treat the symptoms because cough medications are not effective.

5. A 4-year-old girl is brought to the emergency department. She has a "frog-like" croaking sound on inspiration, is agitated, and is drooling. She insists on sitting upright. Which nursing action is the priority for this patient? A. Maintain the airway. B. Obtain a throat culture. C. Start an intravenous line. D. Transport for a chest x-ray.

ANS: A Airway, breathing, and circulation are the primary concerns for all patients. When a patient has acute respiratory distress, the most important nursing intervention is maintaining an airway. Starting an IV is important, but it is not the primary concern. Obtaining a throat culture would be contraindicated at this time, as it could cause vomiting or occlusion of the airway. The child should not be transported for x-ray; it should be done with portable equipment.

3. The pediatric nurse examines a neonate and documents that the baby is pink in color when crying but turns bluish when quiet. Which question would be most appropriate to ask the parent? A. "Does the baby have trouble when eating?" B. "Have your other children had this problem?" C. "How many respiratory infections has she had?" D. "You don't ever prop her bottle, do you?"

ANS: A Choanal atresia is a congenital malformation of the nose in which there is blockage of the posterior side of the nose. It often is associated with bony abnormalities and may affect one side or both sides of the nose. A child with bilateral choanal atresia usually displays respiratory problems during development. A newborn who is pink in color when crying, yet turns bluish when quiet should be suspected of having bilateral choanal atresia or another defect impeding the nasal airway. Another sign of this disorder is feeding difficulties and resultant lack of weight gain. Because this is not genetic, asking about other children with the condition is irrelevant. The number of respiratory infections will not give information as to the possible diagnosis. Because of the potential for aspiration, the baby should be fed in a semi-upright to upright position; however, the wording of the question might make the parent feel defensive.

39. A 3-year-old boy has been hospitalized because he fell down the stairs. His mother is crying and states, "This is all my fault." Which is the nurse's best response? 1. "Accidents happen. You shouldn't blame yourself." 2. "Falls are one of the most common injuries in this age group." 3. "It may be a good idea to put a baby gate on the stairs." 4. "Your son should be proficient at walking down the stairs by now."

2. "Falls are one of the most common injuries in this age group." Falls are one of the most common injuries, and it may make the parent feel better to know that this is common.

What does the therapeutic management of cystic fibrosis (CF) patients include? Select all that apply. 1. Providing a high-protein, high-calorie diet. 2. Providing a high-fat, high-carbohydrate diet. 3. Encouraging exercise. 4. Minimizing pulmonary complication. 5. Encouraging medication compliance.

1, 3, 4, 5. 1. Children with CF have difficulty absorbing nutrients because of the blockage of the pancreatic duct. Pancreatic enzymes cannot reach the duodenum to aid in digestion of food. These children often require up to 150% of the caloric intake of their peers. The nutritional recommendation for CF patients is high-calorie and high-protein. 2. A high-fat, high-carbohydrate diet is not recommended for adequate nutrition. 3. Exercise is effective in helping CF patients clear secretions. 4. Minimizing pulmonary complications is essential to a better outcome for CF patients. Compliance with CPT, nebulizer treatments, and medications are all components of minimizing pulmonary complications. 5. Medication compliance is a necessary part of maintaining pulmonary and gastrointestinal function. TEST-TAKING HINT: The test taker can eliminate answer 2 because patients are not placed on high-fat diets.

19. Which statement by the mother of an 18-month-old would lead the nurse to believe that the child should be referred for further evaluation for developmental delay? 1. "My child is able to stand but is not yet taking steps independently." 2. "My child has a vocabulary of approximately 15 words." 3. "My child is still sucking his thumb." 4. "My child seems to be quite wary of strangers."

1. "My child is able to stand but is not yet taking steps independently." The child should be walking indepen- dently by 15 to 18 months. Because this toddler is 18 months and not walking, a referral should be made for a develop- mental consult. The vocabulary of an 18-month-old should be 10 words or more. Thumb-sucking is still common for 18-month-olds and may actually be at its peak at that age. It is very common for a child of 18 months to exhibit stranger anxiety.

29. A 3-year-old is hospitalized for an ASD repair. The parents have decided to go home for a few hours to spend time with her siblings. The child asks when her mommy and daddy will be back. The nurse's best response is: 1. "Your mommy and daddy will be back after your nap." 2. "Your mommy and daddy will be back at 6:00 p.m." 3. "Your mommy and daddy will be back later this evening." 4. "Your mommy and daddy will be back in 3 hours."

1. "Your mommy and daddy will be back after your nap." Preschoolers understand time in relation to events.

Which child with asthma should the nurse see first? 1. A 12-month-old who has a mild cry, is pale in color, has diminished breath sounds, and has an oxygen saturation of 93%. 2. A 5-year-old who is speaking in complete sentences, is pink in color, is wheezing bilaterally, and has an oxygen saturation of 93%. 3. A 9-year-old who is quiet, is pale in color, and is wheezing bilaterally with an oxygen saturation of 92%. 4. A 16-year-old who is speaking in short sentences, is wheezing, is sitting upright, and has an oxygen saturation of 93%.

1. This child is exhibiting signs of severe asthma. This child should be seen first. The child no longer has wheezes and now has diminished breath signs. TEST-TAKING HINT: The test taker can eliminate answers 2, 3, and 4 by knowing that diminished breath sounds are a sign the patient has a worsening condition. The other bit of information that is essential in this problem is the child's age. The younger the child, the faster the respiratory status can diminish.

32. What can a nurse do to reinforce a 5-year-old's intellectual initiative when he asks about his upcoming surgery? 1. Answer the child's questions about his upcoming surgery in simple terms. 2. Provide the child with a book that has vivid illustrations about his surgery. 3. Tell the child he should wait and ask the doctor his questions. 4. Tell the child that she will answer his questions at a later time.

1. Answer the child's questions about his upcoming surgery in simple terms. The child is taking the initiative to ask questions, as all preschoolers do, and the nurse should always answer those questions as appropriately and accurately as possible.

The parent of a 4-month-old with cystic fibrosis (CF) asks the nurse what time to begin the child's first chest physiotherapy (CPT) each day. Which is the nurse's best response? 1. "Thirty minutes before feeding the child breakfast." 2. "After deep-suctioning the child each morning." 3. "Thirty minutes after feeding the child breakfast." 4. "Only when the child has congestion or coughing."

1. CPT should be done in the morning prior to feeding to avoid the risk of the child vomiting. TEST-TAKING HINT: Answer 4 can be eliminated because of the word "only." There are very few times in health care when an answer will be "only." Answer 3 can be eliminated when one considers the risk of vomiting and aspiration that may occur if percussion is performed following eating.

Who is at the highest priority to receive the flu vaccine? 1. A healthy 8-month-old who attends day care. 2. A 3-year-old who is undergoing chemotherapy. 3. A 7-year-old who attends public school. 4. An 18-year-old who is living in a college dormitory

1. Children between the ages of 6 and 23 months are at the highest risk for having complications as a result of the flu. Their immune systems are not as developed, so they are at a higher risk for influenza-related hospitalizations. TEST-TAKING HINT: The test taker can eliminate answers 2 and 3 by knowing that infants and the elderly are at highest risk for complications related to the flu.

Which is the nurse's best response to the parent of a child diagnosed with epiglottitis who asks what the treatment will be? 1. Complete a course of intravenous antibiotics. 2. Surgery to remove the tonsils. 3. 10 days of aerosolized ribavirin. 4. No intervention.

1. Epiglottitis is bacterial in nature and requires intravenous antibiotics. A 7- to 10-day course of oral antibiotics is usually ordered following the intravenous course of antibiotics. TEST-TAKING HINT: Understanding that epiglottitis is bacterial in nature will lead the test taker to choose the correct answer.

The parent of a child with influenza asks the nurse when the child is most infectious. Which is the nurse's best response? 1. "Twenty-four hours before and after the onset of symptoms." 2. "Twenty-four hours after the onset of symptoms." 3. "One week after the onset of symptoms." 4. "One week before the onset of symptoms."

1. Influenza is most contagious 24 hours before and 24 hours after onset of symptoms. TEST-TAKING HINT: This question requires the test taker to have knowledge of the communicability of influenza.

Which is the nurse's best response to a parent who asks what can be done at home to help a child with upper respiratory infection (URI) symptoms and a fever get better? 1. "Give your child small amounts of fluid every hour to prevent dehydration." 2. "Give your child Robitussin at night to reduce his cough and help him sleep." 3. "Give your child a baby aspirin every 4 to 6 hours to help reduce the fever." 4. "Give your child an over-the-counter cold medicine at night."

1. It is essential that parents ensure their children remain hydrated during a URI. The best way to accomplish this is by giving small amounts of fluid frequently. TEST-TAKING HINT: The test taker can eliminate answers 2 and 4 because overthe- counter cold and cough medications are not recommended for infants.

11. Which finding would the nurse consider abnormal when performing a physical assessment on a 6-month-old? 1. Posterior fontanel is open. 2. Anterior fontanel is open. 3. Beginning signs of tooth eruption. 4. Able to track and follow objects.

1. Posterior fontanel is open. The posterior fontanel should close between 6 and 8 weeks of age. The anterior fontanel usually closes between 12 and 18 months. The infant usually has a first tooth erupt at about 6 months of age. The infant should be able to track objects.

Which statement indicates the parent needs further teaching on how to prevent his other children from contracting respiratory syncytial virus (RSV)? 1. "I should make sure that both my children receive Synagis (palivizumab) injections for the remainder of this year." 2. "I should be sure to keep my infected child away from his brother until he has recovered." 3. "I should insist that all people who come in contact with my children thoroughly wash their hands before playing with them." 4. "I should insist that anyone with a respiratory illness avoid contact with my children until well."

1. Synagis will not help the child who has already contracted the illness. Synagis is an immunization and a method of primary prevention. TEST-TAKING HINT: This question requires the test taker to understand how RSV is transmitted and how to prevent the spread of the virus.

What would the nurse advise the parent of a child with a barky cough that gets worse at night? 1. Take the child outside into the more humid night air for 15 minutes. 2. Take the child to the ER immediately. 3. Give the child an over-the-counter cough suppressant. 4. Give the child warm liquids to soothe the throat.

1. The night air will help decrease subglottic edema, easing the child's respiratory effort. The coughing should diminish significantly, and the child should be able to rest comfortably. If the symptoms do not improve after taking the child outside, the parent should have the child seen by a health-care provider. TEST-TAKING HINT: The test taker must accurately identify that the question is describing a child with croup and know how croup is treated.

Which statement by the parent of a child using an albuterol inhaler leads the nurse to believe that further education is needed on how to administer the medication? 1. "I should administer two quick puffs of the albuterol inhaler using a spacer." 2. "I should always use a spacer when administering the albuterol inhaler." 3. "I should be sure that my child is in an upright position when administering the inhaler." 4. "I should always shake the inhaler before administering a dose."

1. The parent should always give one puff at a time and wait 1 minute before administering the second puff. TEST-TAKING HINT: The test taker evaluates how the parents administer the MDI.

Which child is at highest risk for requiring hospitalization to treat respiratory syncytial virus (RSV)? 1. A 2-month-old who was born at 32 weeks. 2. A 16-month-old with a tracheostomy. 3. A 3-year-old with a congenital heart defect. 4. A 4-year-old who was born at 30 weeks.

1. The younger the child, the greater the risk for developing complications related to RSV. This infant is at highest risk because of age and premature status. TEST-TAKING HINT: The test taker must consider that all of these children have some amount of risk for requiring hospitalization. The 2-month-old has two of the noted risk factors of being premature and a very young infant.

18. The nurse is assessing the pain level in an infant who just had surgery. The infant's parent asks which vital sign changes are expected in a child experiencing pain. The nurse's best response is: 1. "We expect to see a child's heart rate decrease and respiratory rate increase." 2. "We expect to see a child's heart rate and blood pressure decrease." 3. "We expect to see a child's heart rate and blood pressure increase." 4. "We expect to see a child's heart rate increase and blood pressure decrease."

3. "We expect to see a child's heart rate and blood pressure increase."

The parent of a child with cystic fibrosis (CF) is excited about the possibility of the child receiving a double lung transplant. What should the parent understand? 1. The transplant will cure the child of CF and allow the child to lead a long and healthy life. 2. The transplant will not cure the child of CF but will allow the child to have a longer life. 3. The transplant will help to reverse the multisystem damage that has been caused by CF. 4. The transplant will be the child's only chance at surviving long enough to graduate college.

2. A lung transplant does not cure CF, but it does offer the patient an opportunity to live a longer life. The concerns are that, after the lung transplant, the child is at risk for rejection of the new organ and for development of secondary infections because of the immunosuppressive therapy. TEST-TAKING HINT: Answer 4 can be eliminated because of the word "only." There are very few times in health care when an answer will be "only." Answers 1 and 3 can be eliminated if the test taker has a basic knowledge of the pathophysiology of CF.

Which statement about pneumonia is accurate? 1. Pneumonia is most frequently caused by bacterial agents. 2. Children with bacterial pneumonia are usually sicker than children with viral pneumonia. 3. Children with viral pneumonia are usually sicker than those with bacterial pneumonia. 4. Children with viral pneumonia must be treated with a complete course of antibiotics.

2. Children with bacterial pneumonia are usually sicker than children with viral pneumonia. Children with bacterial pneumonia can be treated effectively, but they require a course of antibiotics. TEST-TAKING HINT: The test taker must have an understanding of the differences between viral and bacterial infections.

A school-age child has been diagnosed with strep throat. The parent asks the nurse when the child can return to school. Which is the nurse's best response? 1. "Forty-eight hours after the first documented normal temperature." 2. "Twenty-four hours after the first dose of antibiotics." 3. "Forty-eight hours after the first dose of antibiotics." 4. "Twenty-four hours after the first documented normal temperature."

2. Children with strep throat are no longer contagious 24 hours after initiation of antibiotic therapy. TEST-TAKING HINT: The test taker can eliminate answers 1 and 4 given knowledge of the communicability of strep throat.

50. A 16-year-old male is hospitalized for cystic fibrosis. He will be an inpatient for 2 weeks while he receives IV antibiotics. Which action taken by the nurse will most enhance his psychosocial development? 1. Fax the teen's teacher, and have her send in his homework. 2. Encourage the teen's friends to visit him in the hospital. 3. Encourage the teen's grandparents to visit frequently. 4. Tell the teen he is free to use his phone to call or text friends.

2. Encourage the teen's friends to visit him in the hospital.

33. A 5-year-old boy has always been one of the shortest children in class. His mother tells the school nurse that her husband is 6 tall and she is 57. What should the nurse tell the child's mother? 1. He is expected to grow about 3 inches every year from ages 6 to 9 years. 2. He is expected to grow about 2 inches every year from ages 6 to 9 years. 3. He should be seen by an endocrinologist for growth-hormone injections. 4. His growth should be re-evaluated when he is 7 years old.

2. He is expected to grow about 2 inches every year from ages 6 to 9 years.

How does the nurse interpret the laboratory analysis of a stool sample containing excessive amounts of azotorrhea and steatorrhea in a child with cystic fibrosis (CF)? The values indicate the child is 1. Not compliant with taking her vitamins. 2. Not compliant with taking her enzymes. 3. Eating too many foods high in fat. 4. Eating too many foods high in fiber.

2. If the child were not taking enzymes, the result would be a large amount of undigested food, azotorrhea, and steatorrhea in the stool. Pancreatic ducts in CF patients become clogged with thick mucus that blocks the flow of digestive enzymes from the pancreas to the duodenum. Therefore, patients must take digestive enzymes with all meals and snacks to aid in absorption of nutrients. Often, teens are noncompliant with their medication regimen because they want to be like their peers TEST-TAKING HINT: The test taker needs to understand the pathophysiology of CF and the impact it has on the gastrointestinal system. The test taker also must be familiar with the conditions azotorrhea and steatorrhea.

A 6-week-old is admitted to the hospital with influenza. The child is crying, and the father tells the nurse that his son is hungry. The nurse explains that the baby is not to have anything by mouth. The parent does not understand why the child cannot eat. Which is the nurse's best response to the parent? 1. "We are giving your child intravenous fluids, so there is no need for anything by mouth." 2. "The shorter and narrower airway of infants increases their chances of aspiration so your child should not have anything to eat now." 3. "When your child eats, he burns too many calories; we want to conserve the child's energy." 4. "Your child has too much nasal congestion; if we feed the child by mouth, the distress will likely increase."

2. Infants are at higher risk of aspiration because their airways are shorter and narrower than those of adults. An infant with influenza has lots of nasal secretions and coughs up mucus. With all the secretions, the infant is at an even higher risk of aspiration. TEST-TAKING HINT: The test taker can eliminate answer 1 because it does not give the father an explanation of why his son cannot eat.

26. Which stressor is common in hospitalized toddlers? Select all that apply. 1. Social isolation. 2. Interrupted routine. 3. Sleep disturbances. 4. Self-concept disturbances. 5. Fear of being hurt.

2. Interrupted routine. 3. Sleep disturbances. 5. Fear of being hurt.

12. A mother requests that her child receive the varicella vaccine at the 9-month well-child checkup. The nurse tells the mother that: 1. Children who are vaccinated will likely develop a mild case of the disease. 2. The vaccine cannot be given at that visit. 3. The vaccine will be administered after the physician examines the child. 4. A booster vaccination will be needed at 18 months of age.

2. The vaccine cannot be given at that visit. The nurse should not give the vaccine. The varicella vaccine is not usually administered prior to 1 year of age.

Which child is in the greatest need of emergency medical treatment? 1. 3-year-old who has a barky cough, is afebrile, and has mild intercostal retractions. 2. 6-year-old who has high fever, no spontaneous cough, and frog-like croaking. 3. 7-year-old who has abrupt onset of moderate respiratory distress, a mild fever, and a barky cough. 4. 13-year-old who has a high fever, stridor, and purulent secretions

2. This child has signs and symptoms of epiglottitis and should receive immediate emergency medical treatment. The patient has no spontaneous cough and has a frog-like croaking because of a significant airway obstruction. TEST-TAKING HINT: The test taker must accurately identify that the question is describing a child with epiglottitis. The test taker must also understand that epiglottitis is a pediatric emergency and can cause the child to have complete airway obstruction.

The parent of a child with frequent ear infections asks the nurse if there is anything that can be done to help avoid future ear infections. Which is the nurse's best response? 1. "Your child should be put on a daily dose of Singulair (montelukast)." 2. "Your child should be kept away from tobacco smoke." 3. "Your child should be kept away from other children with otitis media." 4. "Your child should always wear a hat when outside."

2. Tobacco smoke has been proved to increase the incidence of ear infections. The tobacco smoke damages mucociliary function, prolonging the inflammatory process and impeding drainage through the eustachian tube. TEST-TAKING HINT: The test taker can eliminate answer 3 by understanding that otitis media is not a contagious disease process. Answer 4 can be eliminated if the test taker understands that otitis media is not caused by exposing the child to cold air.

Which is the nurse's best response to parents who ask what impact asthma will have on the child's future in sports? 1. "As long as your child takes prescribed asthma medication, the child will be fine." 2. "The earlier a child is diagnosed with asthma, the more significant the symptoms." 3. "The earlier a child is diagnosed with asthma, the better the chance the child has of growing out of the disease." 4. "Your child should avoid playing contact sports and sports that require a lot of running."

2. When a child is diagnosed with asthma at an early age, the child is more likely to have significant symptoms on aging. TEST-TAKING HINT: The test taker can eliminate answer 4 because not all asthmatics also have exercise-induced asthma necessitating use of a fast-acting bronchodilator before playing.

Which assessment is of greatest concern in a 15-month-old? 1. The child is lying down, has moderate retractions, low-grade fever, and nasal congestion. 2. The child is in the tripod position, has diminished breath sounds, and a muffled cough. 3. The child is sitting up and has coarse breath sounds, coughing, and fussiness. 4. The child is restless and crying, has bilateral wheezes, and is feeding poorly.

2. When children are sitting in the tripod position, they are having difficulty breathing. The child is sitting and leaning forward in order to breathe more easily. Diminished breath sounds are indicative of a worsening condition. A muffled cough indicates that the child has some subglottic edema. This child has several signs and symptoms of a worsening respiratory condition. TEST-TAKING HINT: The test taker can eliminate answers 1, 3, and 4 if familiar with common signs and symptoms of respiratory illness.

1. A 6-month-old male is at his well-child checkup. The nurse weighs him, and his mother asks if his weight is normal for his age. The nurse's best response is: 1. "At 6 months his weight should be approximately three times his birth weight." 2. "Each child gains weight at his or her own pace." 3. "At 6 months his weight should be approximately twice his birth weight." 4. "At 6 months a child should weigh about 10 lb more than his or her birth weight."

3. "At 6 months his weight should be approximately twice his birth weight."

6. The nurse is instructing a new breastfeeding mother in the need to provide her pre- mature infant with an adequate source of iron in her diet. Which statement reflects a need for further education of the new mother? 1. "I will use only breast milk or an iron-fortified formula as a source of milk for my baby until she is at least 12 months old." 2. "My baby will need to have iron supplements introduced when she is 4 months old." 3. "I will need to add iron supplements to my baby's diet when she is 2 months old." 4. "When my baby begins to eat solid foods, I should introduce iron-fortified cereals to her diet."

3. "I will need to add iron supplements to my baby's diet when she is 2 months old." Premature infants have iron stores from the mother that last approximately 2 months, so it is important to introduce an iron supplement by 2 months of age. Full-term infants have iron stores that last approximately 4 to 6 months.

16. The nurse is using the FLACC scale to rate the pain level in a 9-month-old. Which is the nurse's best response to the father's question of what the FLACC scale is? 1. "It estimates a child's level of pain utilizing vital sign information." 2. "It estimates a child's level of pain based on parents' perception." 3. "It estimates a child's level of pain utilizing behavioral and physical responses." 4. "It estimates a child's level of pain utilizing a numeric scale from 0 to 5."

3. "It estimates a child's level of pain utilizing behavioral and physical responses." The FLACC scale utilizes behavioral and physical responses of the child to measure the child's level of pain. The scale utilizes facial expression, leg position, activity, intensity of cry, and level of consolability.

17. A 12-month-old boy weighed 8 lb 2 oz at birth. Understanding developmental mile- stones, what should the nurse caring for the child expect the current weight to be? 1. 16lb4oz 2. 20lb5oz 3. 24lb6oz 4. 32lb8oz

3. 24lb6oz Children should triple their birth weight by 12 months of age.

Which is diagnostic for epiglottitis? 1. Blood test. 2. Throat swab. 3. Lateral neck x-ray of the soft tissue. 4. Signs and symptoms.

3. A lateral neck x-ray is a definitive test to diagnose epiglottitis. The child is at risk for complete airway obstruction and should always be accompanied by a nurse to the x-ray department. TEST-TAKING HINT: The test taker can eliminate answers 1, 2, and 4 because epiglottitis is diagnosed by lateral neck films.

Which should the nurse administer to provide quick relief to a child with asthma who is coughing, wheezing, and having difficulty catching her breath? 1. Prednisone. 2. Singulair (montelukast). 3. Albuterol. 4. Flovent (fluticasone).

3. Albuterol is the quick-relief bronchodilator of choice for treating an asthma attack. TEST-TAKING HINT: The test taker must know the medications used to treat asthma and which are used in which situations.

27. Which nursing action would help foster a hospitalized 3-year-old's sense of autonomy? 1. Let the child choose what time to take the oral antibiotics. 2. Allow the child to have a doll for medical play. 3. Allow the child to administer her own dose of Keflex (cephalexin) via oral syringe. 4. Let the child watch age-appropriate videos.

3. Allow the child to administer her own dose of Keflex (cephalexin) via oral syringe. Allowing preschoolers to participate in actions of which they are capable is an excellent way to enhance their autonomy.

36. Which action is a developmentally appropriate method for eliciting a 4-year-old's cooperation in obtaining the blood pressure? 1. Have the child's parents help put on the blood pressure cuff. 2. Tell the child that if he sits still, the blood pressure machine will go quickly. 3. Ask the child if he feels a squeezing of his arm. 4. Tell the child that measuring the blood pressure will not hurt.

3. Ask the child if he feels a squeezing of his arm.

13. What should parents understand is one of the most common causes of injury and death for a 7-month-old infant? 1. Poisoning. 2. Child abuse. 3. Aspiration. 4. Dog bites.

3. Aspiration. Aspiration is a common cause of injury and death among children of this age. These children often find small objects lying on the floor and place them in their mouths. Older siblings are often responsible for leaving small objects around.

Which breathing exercises should the nurse have an asthmatic 3-year-old child do to increase her expiratory phase? 1. Use an incentive spirometer. 2. Breathe into a paper bag. 3. Blow a pinwheel. 4. Take several deep breaths.

3. Blowing a pinwheel is an excellent means of increasing a child's expiratory phase. Play is an effective means of engaging a child in therapeutic activities. Blowing bubbles is another method to increase the child's expiratory phase. TEST-TAKING HINT: The test taker can eliminate answers 1 and 4 because they do not increase the expiratory phase. Play is one of the best ways to engage young children in therapeutic activities.

25. Which comment should the parent of a 2-year-old expect from the toddler about a new baby brother? 1. "When the baby takes a nap, will you play with me?" 2. "Can I play with the baby?" 3. "The baby is so cute. I love him." 4. "It is time to put him away so we can play."

4. "It is time to put him away so we can play." This is a typical statement that would be made by a toddler. Toddlers are very egocentric and do not consider the needs of the other child.

A mother is crying and tells the nurse that she should have brought her son in yesterday when he said his throat was sore. Which is the nurse's best response to this parent whose child is diagnosed with epiglottitis and is in severe distress and in need of intubation? 1. "Children this age rarely get epiglottitis; you should not blame yourself." 2. "It is always better to have your child evaluated at the first sign of illness rather than wait until symptoms worsen." 3. "Epiglottitis is slowly progressive, so early intervention may have decreased the extent of your son's symptoms." 4. "Epiglottitis is rapidly progressive; you could not have predicted his symptoms would worsen so quickly."

4. Epiglottitis is rapidly progressive and cannot be predicted. TEST-TAKING HINT: When something happens to a child, the parents always blame themselves. Telling them epiglottitis is rapidly progressive may be helpful.

A school-age child is admitted to the hospital for a tonsillectomy. During the nurse's post-operative assessment, the child's parent tells the nurse that the child is in pain. Which of the following observations would be of most concern to the nurse? 1. The child's heart rate and blood pressure are elevated. 2. The child complains of having a sore throat. 3. The child is refusing to eat solid foods. 4. The child is swallowing excessively.

4. Excessive swallowing is a sign that the child is swallowing blood. This should be considered a medical emergency, and the physician should be contacted immediately. The child is likely bleeding and will need to return to surgery. TEST-TAKING HINT: Answer 1 can be eliminated if the test taker understands the common vital-sign changes that occur when a person is experiencing pain.

42. Which statement accurately describes how the nurse should approach an 11-year-old to do a physical assessment? 1. Ask the child's parents to remain in the room during the physical exam. 2. Auscultate the heart, lungs, and abdomen first. 3. Explain that the physical exam will not hurt. 4. Explain what the nurse will be doing in basic understandable terms.

4. Explain what the nurse will be doing in basic understandable terms. School-age children are capable of un- derstanding basic functions of the body and can understand what the nurse will be doing if explained in basic terms.

Which physical findings would be of most concern in an infant with respiratory distress? 1. Tachypnea. 2. Mild retractions. 3. Wheezing. 4. Grunting.

4. Grunting is a sign of impending respiratory failure and is a very concerning physical finding. TEST-TAKING HINT: The test taker can eliminate answers 1, 2, and 3 by knowing the signs of respiratory distress. They warrant frequent respiratory assessment, but they are not the most concerning physical signs.

Which would the nurse explain to parents about the inheritance of cystic fibrosis? 1. CF is an autosomal-dominant trait passed on from the child's mother. 2. CF is an autosomal-dominant trait passed on from the child's father. 3. The child of parents who are both carriers of the gene for CF has a 50% chance of acquiring CF. 4. The child of a mother who has CF and a father who is a carrier of the gene for CF has a 50% chance of acquiring CF

4. If the child is born to a parent with CF and the other parent is a carrier, the child has a 50% chance of acquiring the disease and a 50% chance of being a carrier of the disease. TEST-TAKING HINT: Answers 1 and 2 can be eliminated with knowledge of the genetic inheritance of CF. CF is inherited as an autosomal-recessive trait.

The parents of a 5-week-old have just been told that their child has cystic fibrosis (CF). The mother had a sister who died of CF when she was 19 years of age. The parents are sad and ask the nurse about the current projected life expectancy. What is the nurse's best response? 1. "The life expectancy for CF patients has improved significantly in recent years." 2. "Your child might not follow the same course that the mother's sister did." 3. "The physician will come to speak to you about treatment options." 4. The nurse answers their questions briefly, listens to their concerns, and is available later after they've processed the information.

4. The nurse's best intervention is to let the parents express their concerns and fears. The nurse should be available if the parents have any other concerns or questions or if they just need someone with whom to talk. TEST-TAKING HINT: When parents are given information that their child has a chronic life-threatening disease, they are not capable of processing all the information right away; they need time. The parents are often given more information than they can possibly understand and often just need someone to listen to their concerns and needs.

A 2-year-old has just been diagnosed with cystic fibrosis (CF). The parents ask the nurse what early respiratory symptoms they should expect to see in their child. Which is the nurse's best response? 1. "You can expect your child to develop a barrel-shaped chest." 2. "You can expect your child to develop a chronic productive cough." 3. "You can expect your child to develop bronchiectasis." 4. "You can expect your child to develop wheezing respirations."

4. Wheezing respirations and a dry, nonproductive cough are common early symptoms in CF. TEST-TAKING HINT: Answer 2 can be eliminated because of the word "chronic." "Chronic" implies that the disease process is advanced rather than in the initial stages. Answers 1 and 3 can be eliminated if the test taker has knowledge of signs and symptoms of advanced lung disease.

The parent of a child with croup tells the nurse that her other child just had croup and it cleared up in a couple of days without intervention. She asks the nurse why this child is exhibiting worse symptoms and needs to be hospitalized. Which is the nurse's best response? 1. "Some children just react differently to viruses. It is best to treat each child as an individual." 2. "Younger children have wider airways that make it easier for bacteria to enter and colonize." 3. "Younger children have short and wide eustachian tubes, making them more susceptible to respiratory infections." 4. "Children younger than 3 years usually exhibit worse symptoms because their immune systems are not as developed."

4. Younger children have less developed immune systems and usually exhibit worse symptoms than older children. TEST-TAKING HINT: Answer 1 can be eliminated because it does not directly address the mother's question. Answer 2 can be eliminated if the test taker has knowledge of the anatomical structure of a child's airway. Answer 3 can be eliminated because the eustachian tubes have no direct relationship to acquiring croup.

A nurse uses Urie Bronfenbrenner's theory to guide practice. In order to determine a child's microsystem, which items should the nurse assess? (Select all that apply.) A. Dynamics between school and home B. Family C. Peer group D. Parental job status E. School

ANS: B, C, E According to Bronfenbrenner, a child's microsystem includes systems in which the child is actively involved and typically includes family, peer group, and school. The dynamics between school and home make up the mesosystem. A parent's job would be part of the exosystem.

A chloride level greater than _____________________ is a positive diagnostic indicator of cystic fibrosis (CF).

60 mEq/L. The definitive diagnosis of CF is made when a child has a sweat chloride level >60 mEq/L. A normal chloride level is <40 mEq/L. TEST-TAKING HINT: The test taker must have knowledge of tests and normal values used to identify a diagnosis of CF.

The pediatric nurse teaches the mother of a toddler how to choose appropriate toys/activities to stimulate growth and development of her child. Which activities are appropriate for the nurse to include in the teaching session? (Select all that apply.) A. Board games B. Large crayons C. Sliding down a slide D. Tricycle E. Watercolor pencils

ANS: As fine and gross motor skills develop, the toddler enjoys practicing skills such as coloring, turning pages, stacking objects, pushing and pulling toys, molding Play-Doh, running, jumping, and climbing. Around age 3, the toddler may learn to ride a tricycle. The toddler can hold a pencil or a large crayon appropriately and make artwork that is more representative of the object he or she is trying to depict. The toddler does not play interactively with other children; therefore, board games are more appropriate for preschool- and school-age children. Watercolor pencils require fine motor coordination and are more appropriate for a preschool-age child.

A nurse is assessing a 1-year-old child who weighed 7 lb, 8 oz at birth. Today's weight is 23 lb. What conclusion can the nurse make about the child's weight? A. The child is at an expected weight. B. The child is over expected weight. C. The child is seriously overweight. D. The child is seriously underweight.

ANS: A A child's weight should triple by 12 months, so a child born at 7 lb, 8 oz should weigh around lb at 1 year.

A 12-year-old child is in the hospital for an extended period of time. Which action by the nurse would most promote the child's sense of self-esteem? A. Allow the child to set a daily schedule for activities. B. Encourage the child's friend to come and visit. C. Have the child choose food from the menu. D. Let the parents have unlimited visitation time.

ANS: A A schedule enhances normalcy, and allowing the child to participate in setting the schedule will help boost self-esteem. The other actions are all good nursing interventions, but giving the child some control and encouraging participation will help self-esteem.

A nurse observes several preschool-aged children during play and overhears one of them say "My mommy won't let me do that." What conclusion is the most appropriate by the nurse regarding this child's development? A. The child has developed a superego according to Freud. B. The child has mastered Bandura's concept of self-mastery. C. The child is behind in moral reasoning and development. D. The child is in Erikson's autonomy versus shame and doubt phase.

ANS: A According to Freud, between the ages of 3 and 6, children begin to develop a superego, which serves to regulate behavior. The child who knows there are limits to behavior is demonstrating this development. Bandura's concept of self-mastery occurs due to the influence of several factors. This child is too young to have developed this. According to Erikson, the stage of autonomy versus shame and doubt typically occurs between the ages of 1 and 3. The child is also too young to have mastered the tasks involved in moral reasoning, and so one cannot say he is behind.

25. A child has been diagnosed with a localized herpes simplex virus (HSV) type 1 infection. The nurse is educating the parents on topical acyclovir (Zovirax) ointment. Which statement by the nurse is most appropriate to include during the medication teaching session? A. "Acyclovir can shorten the outbreak." B. "If this doesn't work we can give it IV." C. "This medication will cure the infection." D. "Zovirax must be used for the child's life."

ANS: A Acyclovir and penciclovir (Danavir) can be used to shorten the duration and lessen the pain of HSV infection. It is not curative. IV medication is used for disseminated infection or in children with severe immunocompromise. The medication is used during outbreaks.

A nurse educator is presenting developmental theories to a group of students. Which statement encompasses the impact that Carol Gilligan had on developmental theories? A. Helped define moral development in women as unique from men B. Described the seven stages of religious/faith development C. Identified nine temperamental traits present at birth in all children D. Viewed the status of the family's oldest child as the marker for transition

ANS: A Carol Gilligan's work focused on the differences in moral development in women. The seven stages of faith/religious development was the work of James Fowler. The nine temperamental traits was the work of Thomas, Chess, and Birch. The family development theory that uses the oldest child as the marker for transition within a family is the work of Duvall.

1. A nursing faculty member is explaining the pediatric immune system to students. Which statement is correct? A. Children are born with intact immune systems. B. Children's immune systems develop over 1 year. C. Immunity isn't functional until about 6 months. D. Mothers' immunity is babies' primary line of defense.

ANS: A Children are born with an intact immune system. There immune system, however, is immature. Infants do retain some immunity from their mothers from birth until about 6 months. The other statements are incorrect.

A nursing manager wants to decrease the amount of stress children have during hospitalization. What environmental change can the manager implement to best meet this goal? A. Create a treatment room for procedures. B. Have dim lighting installed in patient rooms. C. Keep the play area unlocked and open at all hours. D. Provide guest trays for parents staying in the room.

ANS: A One way to decrease the stress of hospitalization for children is to make the child's room a safe place where painful and frightening treatments and procedures do not occur. The manager should create a treatment room on the unit. Dim lighting might be peaceful and lead to better rest, but is not the best answer, nor is providing guest trays. Play areas for younger children especially should be kept secure for patient safety.

4. The pediatric nurse is assessing a child who complains of a stomach ache. This is the child's fourth visit to the clinic for the same problem, and there has not been a diagnosis yet. What action by the nurse is best? A. Ask about child's functioning and development. B. Instruct the mother not to keep the child home from school. C. Obtain a thorough family history from the mother. D. Perform an abdominal exam and document findings.

ANS: A Children with anxiety often present with somatic complaints that do not have a physical cause. The nurse should inquire about the symptoms characteristic of all anxiety disorders, including impaired function or development related to anxiety. The nurse should not just instruct the mother to send the child to school, as this is not a helpful strategy in alleviating the problem. A thorough history is important, but most likely would have already been done in prior visits. Performing and documenting an abdominal exam are both important, but again will not provide specific assistance in this situation.

38. The day after attending a large birthday party for a classmate, a child breaks out in a rash characteristic of chickenpox. When counseling the parents, which information is most appropriate? A. "Inform all the parents of children at the party that your child has chickenpox." B. "This disease is spread through respiratory droplets, so don't get too close." C. "We can give your child a dose of varicella zoster immune globulin right away." D. "Your child is only contagious for 3 days after the rash first appears."

ANS: A Children with chickenpox are contagious from 1-2 days prior to the rash erupting until the time when all the lesions have crusted over, usually about 7 days. The parents of this child should inform the other parents about their children's exposure to the disease. The disease is spread via airborne and contact routes. Immune globulin can be used within 72 hours after an exposure in immunosuppressed children.

A nurse is observing infants at a day-care center to determine how their behavior fits into attachment theories. The nurse notes a wide variety of attachment behaviors. What other assessment is appropriate for the nurse to make? A. Cultural background B. Fussiness of the babies C. Time dropped off D. Time spent in day care

ANS: A Cultural background plays an important role in behavior, including demonstrations of attachment. Infants will show attachment in a manner consistent with their culture. The nurse should note the cultural background of the observed infants. The other information may be useful, but is not as important in judging attachment as is cultural background.

17. The parents of a child diagnosed with cystic fibrosis (CF) consult the nurse, stating they want to have more children but are worried about subsequent children also having the disease. Which information does the nurse provide the parents? A. "Each child has a 25% chance of inheriting the disease." B. "This disease is rare, so other children should not be affected." C. "Unfortunately, there is no way to predict if they will have it." D. "You should have genetic testing to see who the carrier is."

ANS: A Cystic fibrosis is an inherited autosomal recessive disorder. When both parents carry the defective gene, each child has a 25% chance of inheriting the defective gene from both parents and manifesting the disease. It is a common disorder, especially in Caucasians, affecting 1 in 3,000 live births.

During a well-child visit, the pediatric nurse assesses a 2-year-old child for language development. Which developmental domain is the nurse assessing? A. Cognitive B. Family development C. Moral/spiritual D. Psychosocial

ANS: A Developmental domain refers to a way of understanding the total child in relation to the mind, body, and spirit. These domains include: physical, psychosocial (emotional, psychological, and social), cognitive (including language and intelligence), moral/spiritual, and family development.

The pediatric nurse teaches the parents of a 3-month-old baby the principles of growth and development that will occur in their child's lifetime. Which statement accurately describes one of these principles? A. Each child progresses through predictable stages within a predictable timeframe. B. Growth and development begin in infancy and continue until the adult years. C. Growth refers to the ongoing process of adapting throughout the life span. D. Within each child, body systems develop at the same rate.

ANS: A Growth refers to the continuous adjustment in the size of the child internally and externally. Development, on the other hand, refers to the ongoing process of adapting throughout the life span. Growth and development are continuous processes from conception to death. Although development advances in an orderly sequence, each child progresses through the predictable stages within a predictable timeframe. The established guidelines are only guidelines that have been developed through observation over time, and within each individual body systems develop at differing rates.

33. The pediatric nurse explains to a nursing student about the most important role the nurse has in preventing disease. What does this role include? A. Ensuring that immunizations are up to date in all children B. Facilitating research on new forms of immunizations C. Giving reminders about immunizations to parents in clinic D. Scheduling and conducting immunization clinics

ANS: A Immunizations are the cornerstone of communicable disease prevention. The most important role the nurse has related to this topic is to ensure that all children in contact with him or her have vaccinations that are up to date. The other activities can be important components of disease prevention, but are not as important.

What important safety measure regarding the use of insect repellents does the nurse teach a parenting group? A. Apply repellents to the child's clothing. B. There are no substitutes for DEET. C. Use oil of lemon eucalyptus on babies. D. Wash repellent off after 2 hours.

ANS: A Insect repellents are harsh chemicals and should be applied to the clothing of children. Spraying the product on the adult's hands and then spreading it on the face and head and exposed skin is an acceptable ways of applying insecticide to a limited amount of skin. The CDC has recently recognized some insecticides that do not contain DEET. Oil of lemon eucalyptus is safe on children over the age of 3. Although insecticides should not stay on for long periods of time, there is no strict 2-hour limit

1. A nurse wishes to increase an at-risk child's resilience. What action by the nurse would be most helpful? A. Assist the child in recognizing internal resources. B. Help the child see that others are worse off. C. Instruct the child to "toughen up." D. Teach the child therapeutic communication skills.

ANS: A Resilience is the ability to recognize and use internal or external resources in dealing with adversity. Helping the child see his or her internal resources is a positive step in improving resiliency. Telling the child that others are worse off than he or she is or that he or she should "toughen up" is dismissive and not helpful. Teaching appropriate communication is important, but children do not need therapeutic communication skills.

A family is raising a child with a serious disability, and the caregivers appear stressed and short tempered during well-child visits. What does the nurse understand about the emotional impact of this situation on the family? (Select all that apply.) A. Caregiver fatigue is a constant threat due to ongoing medical needs. B. Confusion and distress occur because they don't have the child they expected. C. Family routine is disrupted by the constant needs of the disabled child. D. Siblings adapt well to having a disabled child and enjoy helping out. E. The family may worry about possible painful procedures and upsetting the child.

ANS: A, B, C, E The family members of a disabled child are beset with emotional upset and confusion about the realities they face with their disabled child. They mourn because they did not get the child they expected, and they can be distressed by the child's pain and need for ongoing medical treatments. Caregiver fatigue is a real possibility, and normal routines are disrupted. Siblings can adapt to the disabled child and may enjoy helping out, but this is not a typical finding and should not be expected.

The pediatric clinic nurse cares for many children with viral upper respiratory infections. Which seasonal characteristics of these infections would the clinic nurse include when educating parents? (Select all that apply.) A. Adenovirus is very prevalent in the winter. B. Coronavirus is commonly seen in the spring. C. Infections with parainfluenza are seen in autumn. D. Respiratory syncytial virus is a problem in summer. E. Rhinovirus is more common in the early fall.

ANS: A, B, C, E The seasonal nature of common viruses leading to respiratory infections includes the following correlations: adenovirus is common in winter and spring, coronavirus is common in winter and spring, parainfluenza is common in winter, respiratory syncytial virus is common in winter and spring, and rhinovirus is common in early fall and late spring.

The pediatric clinic nurse is teaching a parent home-care measures for a 12-year-old child diagnosed with viral pharyngitis. Which information is appropriate for the nurse to include in the teaching session? (Select all that apply.) A. Bedrest for a couple days will help recovery. B. Have the child gargle with warm saline. C. Make sure your child continues to eat. D. Offer fluids in small amounts, frequently. E. Warm liquids are better tolerated than cold.

ANS: A, B, D Children with pharyngitis will recover faster if they stay in bed while acutely ill. An older child can be taught to gargle with warm saline and to use throat lozenges. Cool, bland liquids are usually tolerated better than warm or hot liquids and solid food. The child should not be forced to eat while it is painful, but should stay hydrated.

The nursing student reads the diagnosis of a lower respiratory infection in a patient's chart. The student knows that the lower respiratory system is composed of which structures? (Select all that apply.) A. Alveoli B. Bronchi C. Larynx D. Lungs E. Pharynx

ANS: A, B, D The lower respiratory tract is composed of the lungs, bronchi, bronchioles, and alveoli. The larynx and pharynx are part of the upper respiratory system.

The nurse is providing discharge instructions to the parents of a 3-year-old going home after minor surgery. What information should the nurse include? (Select all that apply.) A. Actions and side effects of prescribed medication B. Activity that is expected and encouraged C. How to take the child's rectal temperature D. Signs and symptoms of infection E. When the child can eat and drink again

ANS: A, B, D, E Discharge teaching after minor surgery involves many items, including actions and side effects of prescribed medications; activity that is allowed, expected, and should be encouraged; signs and symptoms of infection; and when the child can eat and drink again. Usually, temperature on children is taken via the axillary route.

A nurse is providing anticipatory guidance to parents of a toddler. Which objects does the nurse include as a frequent cause of aspiration? (Select all that apply.) A. Balloons B. Hot dog bits C. Licorice sticks D. Peanuts E. Popcorn

ANS: A, B, D, E Small, often roundish, objects such as peanuts, popcorn, hot dog bits, vegetable pieces, gel fruit snacks, balloons, coins, pen tops, button eyes, small toy parts, and pins are often aspirated.

The pediatric nurse is providing care to a neonate diagnosed with cystic fibrosis. When discussing the clinical manifestations of this disease process, which topics will the nurse include in the teaching session? (Select all that apply.) A. Anemia B. Malnutrition C. Scant, hard stools D. Meconium ileus E. Rectal prolapse

ANS: A, B, D, E The initial presentation of cystic fibrosis in the neonate appears in the gastrointestinal system. The newborn may have a meconium ileus, with meconium so thick that it causes obstruction and requires surgical removal. The infant may initially have bulky stools that are frothy and foul-smelling. Prolapse of the rectum may also occur in infancy and childhood. Malnutrition, anemia, and growth failure persists despite normal caloric intake.

The child development specialist explains to the nursing students that therapeutic play has several functions, including which of the following? (Select all that apply.) A. Allows children to demonstrate their emotions B. Decreases stress associated with hospitalization C. Gives parents a break from watching their child D. Helps correct children's misconceptions about care E. May help child learn new coping skills

ANS: A, B, D, E Therapeutic play has several purposes, including allowing children to demonstrate their emotions, decreasing stress, helping to correct misconceptions about treatments, and helping the child learn to cope with and master stressful situations. A purpose is not to give the parents a break.

A nurse is providing anticipatory guidance to the parents of a school-age child. What information is appropriate for the nurse to include for school-age children? (Select all that apply.) A. Discuss physical changes related to puberty. B. Discuss smoking and substance abuse. C. Provide simple explanations for questions. D. Read short stories daily to help language. E. School-age children require 8 to 12 hours of sleep each night.

ANS: A, B, E Appropriate anticipatory guidance for a school-age child is to discuss physical changes that will occur in puberty (before they happen), discuss smoking and substance abuse (hopefully before they are exposed to it), and inform parents that children this age still need 8 to 12 hours of sleep a night. Simple explanations and short stories are more appropriate for toddlers.

The nurse working in a pediatric burn unit explains to new registered nurses that which is the most common cause of death in burned children? A. Hypovolemic shock B. Infection C. Sepsis D. Thrombotic events

ANS: B About 75% of burn mortality is related to infection.

A nurse is providing anticipatory guidance to the parents of a 5-month-old baby. Which nursing statements are appropriate by the nurse to these parents? (Select all that apply.) A. "Do not leave the child alone on the changing table." B. "Until the age of 3, falls are common due to large head size." C. "Peek-a-boo is an appropriate game for this age." D. "Wrap up mini-blind ties so the child can't reach them." E. "You need to childproof all your cabinets now."

ANS: A, C An infant begins to roll over by the age of 6 months and could easily fall off a changing table with this newfound skill. Infants respond strongly to peek-a-boo games due to the development of object permanence. Head size matches torso size by about 1 year of age. Wrapping up mini-blind cords and childproofing cabinets can be done at any time, but those are activities more appropriate for the family with an increasingly mobile toddler.

The nurse is caring for a child who experiences frequent ear infections. The child's mother wants to know why this is occurring. Which anatomical differences in the pediatric patient increase the risk for otitis media? (Select all that apply.) A. Impaired drainage B. Longer, thinner eustachian tubes C. Shorter, horizontal eustachian tubes D. Typical lying-down position of infants E. Underdeveloped cartilage lining

ANS: A, C, D, E The following factors lead to otitis media in children: the eustachian tubes are short, wide, and straight and lie in a horizontal plane; the cartilage lining is undeveloped, making the tubes more distensible; the normally abundant pharyngeal lymphoid tissue readily obstructs the eustachian tube openings in the nasopharynx; immature humoral defense mechanisms increase the risk of infections; and the lying-down position of infants favors the pooling of fluid, such as formula, in the pharyngeal cavity.

A parent brings a 2-year-old child to the clinic, reporting that the child has an ear infection. Which assessment information leads the nurse to suspect a diagnosis of bacterial otitis media? (Select all that apply.) A. Acute otalgia B. Dull, throbbing pain C. Fever of 104°F (40°C) D. High-pitched crying E. Poor feeding F. Rubbing the ear

ANS: A, C, E, F Manifestations of acute otitis media include otalgia, irritability, otorrhea, fever that may be high, poor feeding, rubbing or pulling at the ear, bulging tympanic membrane, enlarged lymph glands, and visualization of purulent material during an otoscopic exam.

A nurse is working with a child who has a chronic illness requiring medication and frequent hospitalizations. The parents refuse to allow the child to go to the playroom, fearing germs from other children will harm their child. Which actions by the nurse are most appropriate? (Select all that apply.) A. Allow the parents to voice their concerns without being judgmental. B. Consult with a child developmental specialist for appropriate in-room activities. C. Document the parents' choice on the patient record and inform the staff. D. Help the parents see the ways their child is normal while having an illness. E. Tell the parents that the playroom and all its objects are disinfected often.

ANS: A, D The nurse should first listen to the parents' concerns in an open, nonjudgmental way to promote communication. Parents of ill children should be helped to see their child as a child with an illness, rather than a sick child. This viewpoint allows the child more freedom to participate in age-appropriate activities that stimulate growth and development and a sense of self-esteem. If this approach is not successful, a consultation for in-room activities would be beneficial. Of course the outcome should be noted on the chart, but this alone does not help this problem. Educating the parents about the ways in which infection control is practiced in the playroom is a helpful strategy, but this in itself is insufficient and dismissive of their concerns.

The pediatric clinic nurse assesses a child who reports swallowing problems and skin changes on the hands in response to cold exposure. Which other manifestations will the nurses assess for in this child? (Select all that apply.) A. Calcinosis B. Fungal nail infections C. Oral thrush D. Sclerodactyly E. Telangiectasias

ANS: A, D, E This child has two manifestations referred to as CREST syndrome (Raynaud's phenomenon and esophageal dysmotility). The other three signs are calcinosis (formation of calcium deposits under the skin), sclerodactyly (stiff skin over the hands), and telangiectasias (tiny broken capillaries on skin). Fungal nail infections and oral thrush are not related.

A 10-year-old child who has been hospitalized frequently and for long periods of time has the nursing diagnosis of delayed growth and development. Which action by the child would demonstrate that outcomes for this diagnosis have been met? A. Able to play harmoniously with peers B. Does own homework independently C. Seeks out parental approval for activities D. Selects age-appropriate games and toys

ANS: B A 10-year-old child is in the Erikson stage of industry versus inferiority. Mastery of tasks leads to self-confidence. Industry is apparent when the child feels capable of doing homework or other assigned tasks independently. This shows resolution of the nursing diagnosis, as appropriate developmental tasks have been accomplished. The other actions do not show resolution of this task.

A parent is frustrated that her toddler cannot button a shirt on his own. Which teaching point is most appropriate for the nurse to provide to this parent? A. A toddler is incapable of buttoning his own shirt, and the parent should stop pushing. B. Developing large muscle groups has to occur before developing small muscle groups. C. The parent should select toys and games that will help the child develop and master this skill. D. The child has not met a major milestone and needs a developmental consultation.

ANS: B A concept in development is that gross motor skills must be developed first and used as the foundation for fine motor skills. The larger muscle groups develop first. Buttoning a shirt requires fine motor skills that this child has not yet mastered. It is too early to expect this activity from the child. Telling the parent to stop pushing the child is disrespectful and does not teach the parent information related to growth and development. Appropriate games and toys can help children with growth and development, but this child is too young to master the skill. The child does not need a developmental consultation.

A nurse needs to administer medication to a toddler. What action by the nurse is most likely to gain cooperation from the child? A. Allow the child to negotiate a "reward." B. Allow the parent to give the medication. C. Explain that medicine is not a punishment. D. Let the toddler self-administer the medicine.

ANS: B A toddler may consider medicine to be a punishment and may resist taking it. Because the parent is a comforting figure for him or her, allowing the parent to administer the medication is a good option for the uncooperative child. Telling a child of this age that medicine is not a punishment is a good idea, but will not be as successful as letting the parent give it. Negotiation is the hallmark of the school-age child. Preschoolers may be able to self-administer medication with close supervision.

A parent is frustrated that her toddler wants to do everything on his own and in "my way." The parent wants to know the appropriate way to discipline the child for not obeying and allowing the parent to dress him quickly in the morning. Which response by the nurse is most appropriate? A. "At this stage in life, discipline is not very effective and will frustrate you both more." B. "I know it's frustrating, but being independent is a very important job at this age." C. "Put him in a time-out, and because he is 2 years old, have him in time-out for 2 minutes." D. "You really need to allow your child to be independent as much as possible."

ANS: B According to Erikson's theory, between 1 and 3 years of age, children are in the stage of autonomy versus shame and doubt. It is the time for the child to establish willpower, determination, and a can-do attitude about self. Discipline can be effective at this age. The parent should not be instructed to put the child in time-out for developmentally appropriate behavior. Simply telling the parent she should let the child be independent does nothing to reduce frustration, as the parent does not know the rationale behind it.

A couple brings their child to the well-child clinic for guidance in improving the child's school performance. The nurse assesses the child and finds that favorite activities are running, playing basketball, and building models. Which conclusion is most appropriate for the nurse to make based on the assessment findings? A. Hyperactive and may need medication B. Learns best through bodily activity C. Needs firm structure for completing schoolwork D. Normal activity; school performance will improve with time

ANS: B According to Howard Gardner, there are eight forms of intelligence: bodily-kinesthetic, interpersonal, intrapersonal, linguistic, logical-mathematical, musical, naturalistic, and spatial. Although children possess all eight forms, they typically develop one to a greater degree. Using this form of intelligence is best to help the child learn. By assessing the child's hobbies and interests, the nurse gains insight into which form a child uses. This child prefers the bodily-kinesthetic form of intelligence, and using bodily motion in learning activities will improve learning. The other answers are not accurate.

A nurse has assessed a 14-year-old hospitalized patient over several days and notes that the child has difficulty with abstract concepts and is unable to appreciate diverse points of view. According to Piaget, how would the nurse categorize this child's development? A. Ahead in development B. Behind in development C. Development is normal D. Not applicable to development

ANS: B According to Piaget, children age 11 to 15 should be in the formal operational stage, in which they are able to use abstract reasoning and can consider both sides of an issue.

21. A nurse is working with a student on the pediatric unit caring for patients in contact isolation for infectious diarrhea. Which action by the student warrants intervention by the nurse? A. Changes gloves, performs hand hygiene after touching a contaminated site B. Performs hand hygiene with alcohol-based rubs after caring for patients C. Uses an alcohol-based hand sanitizer prior to putting on gloves D. Washes her hands with soap and hot water when they are visibly soiled

ANS: B After caring for patients with potential or actual infectious diarrhea, hand hygiene is performed using soap and hot water. The other actions are correct.

A student is learning about cutaneous skin reactions. Which description is correct? A. Blistering: heat-related B. Exanthema: an eruption C. Pustular: contagious D. Urticarial: round patches

ANS: B An examthema is an eruption. Blistering is swelling. Pustular is raised and filled with white blood cells. Urticarial is itching.

A pediatric nurse works with the family of twins at designated stages in their lives to help the parents anticipate and move through the periods of disequilibrium. What is the model of child development that focuses on disruptive periods of development? A. Bowlby's attachment theory of development B. Brazelton's touchpoints model of development C. Erikson's stages of development D. Freud's stages of psychosexual development

ANS: B Brazelton described touchpoints as "periods during the first 3 years of life during which children's spurts in development resulted in pronounced disruption in the family system." The touchpoints perspective assumes, among other things, that parents know their children better than they know anyone else, and with that in mind, the nurse works with the family at the various touchpoints to help them anticipate and move through the periods of disequilibrium. Bowlby focuses on the impact of separation from mothers seen in infants. Erickson's theory focuses on the influence of social interaction on development. Freud's theory regards the impact of psychosexual instincts as most important in development.

The pediatric nurse assessing a patient for breath sounds documents a loud, high-pitched sound heard only over the trachea. The nurse should document this finding as which of the following? A. Adventitious breath sound B. Bronchial breath sound C. Bronchovesicular breath sound D. Vesicular breath sound

ANS: B Bronchial breath sounds are loud, high-pitched, and heard only over the trachea. Bronchovesicular breath sounds are of intermediate intensity and pitch, with equal inspiratory and expiratory phases. These sounds are best heard between the scapulae and over the mainstem bronchi. Vesicular breath sounds are heard throughout the lung fields. These soft and low-pitched sounds have a longer inspiratory than expiratory phase. Adventitious sounds of these three classifications are described as crackles, wheezes, and rhonchi, respectively.

A parent of a teething child asks for guidance on nonpharmacological treatments for gum pain. What herbal preparation can the nurse suggest? A. Aloe vera B. Chamomile C. Echinacea D. Tea tree oil

ANS: B Chamomile is used for the pain of teething, colic, and stomach aches. Aloe vera is used orally for constipation and topically for minor skin irritation. Echinacea is used for colds, fever, and inflammation of the mouth and pharynx. Tea tree oil is a topical treatment for skin infections.

5. A 4-month old baby was recently hospitalized with septicemia and now has a severe diaper rash. Which primary immunodeficiency disorder does the nurse suspect? A. Antibody deficiency: B-cell disorder B. Combined deficiency: T- and B-cell disorder C. Complement defect disorder D. Phagocyte defect disorder

ANS: B Combined deficiency: T- and B-cell disorder usually manifests before 6 months of age and includes severe infections such as meningitis and septicemia, diaper dermatitis, and opportunistic infections. Antibody deficiencies are usually seen after 6 months of age. Complement defect disorder is often accompanied by autoimmune diseases. Phagocyte defect disorders include impetigo, mouth ulcers, suppurative adenitis, osteomyelitis, and poor wound healing.

A 7-year-old hospitalized for a fracture following a car crash tells the pediatric nurse "God is in heaven with his angels and is looking down on me." Which stage of Fowler's spiritual development is this child exhibiting? A. Intuitive-projective stage B. Mythical-literal stage C. Synthetic-convention stage D. Undifferentiated stage

ANS: B In stage 0—undifferentiated (infancy)—the infant is learning a "fund of basic trust and the relational experience of mutuality with the one(s) providing primary love and care." In stage 1—intuitive-projective (ages 2 to 6 or 7)—beliefs and faith are unquestioning. It is a time of fantasy and magical thinking. In stage 2—mythical-literal (ages 6 to 11)—the child retells the spiritual stories and takes them literally and concretely. In stage 3—synthetic-convention (typically begins around 12 or 13 years)—the young person begins to personalize beliefs.

A pediatric nurse examines a 7-year-old at a well-child visit. Based on Erikson's theory, which basic task does the nurse anticipate for this child? A. Balance independence and self-sufficiency against uncertainty and misgiving. B. Develop a sense of confidence through mastery of different tasks. C. Develop resourcefulness to achieve and learn new things without self-reproach. D. Recognize there are people in his or her life who can be trusted to take care of basic needs.

ANS: B In the trust vs. mistrust stage (birth to 1 year), the task is for the child to recognize that there are people in his or her life (parents) who can be trusted to take care of his or her basic needs. In the autonomy vs. shame and doubt stage (1 to 3 years), the task is for the child to balance independence and self-sufficiency against the predictable sense of uncertainty and misgiving when placed in life's situations. In the initiative vs. guilt stage (3 to 6 years), the child's task is to develop resourcefulness to achieve and learn new things without receiving self-reproach. In the accomplishment/industry vs. inferiority stage (6 to 12 years), the child develops a sense of confidence through mastery of tasks.

A middle-aged woman has brought a fussy baby to the pediatric clinic. After placing the woman and child in an exam room, which of the following questions should the nurse ask first? A. "Have you taken the baby's temperature?" B. "How are you related to the baby?" C. "How long has the baby been so fussy?" D. "What brings you to the office today?"

ANS: B It is crucial to establish the relationship between a child and the adult who brings the child in for treatment. The nurse should ascertain the woman's identity; it is possible she is not legally able to provide consent for treatment. The other questions are important assessment questions, but establishing the identity of the adult comes first.

A mother brings her child to the well-child clinic. She is distraught because the child tested well below normal on a school-administered intelligence (IQ) test. Which action by the nurse is the most appropriate? A. Advise the mother to have the test repeated next year. B. Assess cultural background and economic status. C. Facilitate a referral to a developmental specialist. D. Reassure the mother that IQ tests are often wrong.

ANS: B Many IQ tests have been criticized for not considering the impact of culture or socioeconomic status. The nurse should assess these factors before proceeding further. Simply reassuring the mother that IQ tests are often wrong does nothing to help her with her concern and sounds paternalistic. That statement could also cause confusion; as the school administered the test, the mother might not believe the nurse.

A 1-week-old infant is in the pediatric clinic. The birth weight was 8 lb, 1 oz (3.65 kg). Today the infant weighs 7 lb (3.17 kg). The mother breastfeeds exclusively. What action by the nurse is best? A. Assess the mother's breastfeeding technique. B. Document the finding and alert the provider. C. Reassure the woman that weight loss is normal. D. Refer the mother to a lactation consultant.

ANS: B Newborns often lose 5-10% of their birth weight during the first week of life. However, a weight loss greater than 10% needs further evaluation. This baby has lost just over 10% of birth weight. The nurse should document the finding and alert the health-care provider. Assessing breastfeeding technique and referral to a lactation consultant may be appropriate depending on the etiology of the problem. The nurse should not reassure the mother that the weight loss is normal because it is excessive for the first week of life.

A teenager is in the family practice clinic for a school physical. When the parent leaves the room, the teen admits to "cutting myself" after a relative "touched me in my private area." What action by the nurse is most appropriate? A. Document the statements and alert the provider. B. Explain that this information must be shared. C. Have the secretary call the police department. D. Reassure the teen of confidentiality rules.

ANS: B Older children (teens and preteens) often prefer to be interviewed in private where concerns of a personal nature can be shared in a safe area. Information about sexuality is often discussed at this time. The nurse is responsible for maintaining privacy except in situations of abuse or where a life-threatening situation exists. The nurse should first explain to the teen that this information needs to be shared with the parent before doing anything else. The information does need to be documented and the provider alerted, but not as the priority. Each facility will have policies in place to report possible abuse, which may or may not involve calling the police, Child Protective Services, or a social worker. However, reporting will wait until the nurse explains to the teen that the information needs to be shared.

A mother has brought her 3-year-old child in to the clinic over concern about the child's lack of development in the last 3 months. Which information is the most appropriate for the nurse to provide to this mother? A. Children should continue their growth and development uninterrupted. B. Periods of growth and development are often followed by periods of rest. C. There is no need for concern unless no changes are seen for 1 year. D. A 3-year-old often does not exhibit changes in growth and development.

ANS: B Periods of growth and development are often followed by a period of "rest." The periods of rest allow the child to incorporate the new growth or the newly developed skill into his or her personal repertoire more completely before attempting the next level. The other statements are in accurate.

A nurse is listening to a school-age child's heart sounds and hears an abnormal noise after S2 that is heard best when the child is lying in the left lateral position. What action by the nurse is most appropriate? A. Arrange a cardiology consult. B. Document the findings in the chart. C. Notify the provider immediately. D. Perform assessments for fluid balance.

ANS: B This sound is an S3 and is often heard in children and young adults. It is usually a benign finding but should be documented in the chart. A cardiology consult is not needed, nor does the nurse need to notify the provider immediately. Because this sound is not usually associated with heart disease, the nurse does not need to perform an assessment of fluid balance.

A 15-year-old adolescent tells the school nurse that he is busy preparing for a forensics competition and would like to be a lawyer one day. According to Piaget's stages of development, the adolescent is in which stage of cognitive development? A. Concrete operational stage B. Formal operational stage C. Preoperational stage D. Sensorimotor stage

ANS: B Piaget's final stage of cognitive development is the formal operational stage, during which the 11- to 15-year-old child uses abstract reasoning to handle difficult concepts and to analyze both sides of an issue. In the concrete operational stage, the 7- to 11-year-old child is much more able to organize thought in a logical order. The child is able to categorize and label objects. It is also possible at this stage for the child to solve concrete problems. In the preoperational stage, the child is still not capable of logical thinking, but due to an increased ability to use words and actions together, the child is increasingly able to connect cognitively with the world. The sensorimotor stage is the initial stage, from birth to age 2, in which primary cognition takes place through the senses.

45. A child is admitted after a severe asthma attack. Respiratory status is stable at this time. Which is the priority intervention for this child? A. Determining the trigger B. Ensuring hydration C. Promoting rest D. Providing support

ANS: B Priorities in any emergency are airway, breathing, and circulation. Because the respiratory status is now stable, attention is placed on ensuring that the child is hydrated. This is important for two reasons: first, hydration helps liquefy secretions; second, the mouth-breathing child will have lost a great deal of fluid through insensible loss, which needs to be replaced. All other options are important parts of care of this child; however, hydration is the priority.

A nurse wishes to prevent radiant heat loss from an infant. What action is the most appropriate? A. Apply warm blankets to the baby. B. Cover the baby's head with a cap. C. Place a space heater near the baby. D. Warm and humidify the oxygen.

ANS: B Radiant heat loss occurs as warmth dissipates into the environment, often occurring rapidly and accounting for over 50% of infant heat loss. The nurse covers the baby's head with a cap. Warm blankets would warm the baby through conduction. A space heater provides convection. Breathing warmed oxygen is preventing heat loss through respiration.

An adolescent patient has acne characterized by pustules and scarring. Which teaching information is most appropriate for the nurse to give the patient and family? A. "Scrub your face vigorously twice a day with soap and water." B. "Tetracycline (Sumacin) will make you more sensitive to the sun." C. "Use good moisturizer because treatment is very drying." D. "You need a pregnancy test every 3 months on isotretinoin (Accutane)."

ANS: B Tetracycline, minocyline, and doxycycline all cause photosensitivity, so the nurse should warn the teen about sun exposure and use of tanning beds. Skin cleansing should be gentle. Although acne treatments are very drying, makeup and moisturizers should not be applied on top of the dry skin. Pregnancy tests are done prior to starting treatment with Accutane, every month during treatment, and 1 month after treatment has stopped.

The pediatric nurse is assessing a 5-year-old for developmental milestones. Which assessment tool should the nurse use? A. CHEOPS scale B. Denver II screening tool C. FLACC scale D. OLD CAT questions

ANS: B The Denver II assesses personal-social, fine motor-adaptive, gross motor, and language skills to gauge performance on developmental milestones. The CHEOPS and FLACC scales are used to assess pain. The mnemonic OLD CAT is used to obtain a patient's pain history and includes questions on onset, location, duration, character, aggravating and alleviating factors, and timing.

A nursing manager is concerned about frequent errors on the pediatric unit and wants to decrease them. What action by the manager is best? A. Have two nurses verify all new orders when they are written. B. Institute a standardized handoff format at shift change. C. Provide remedial education to nurses who make errors. D. Require charge nurses to verify care plans with staff nurses.

ANS: B The Joint Commission has identified handoff communication as contributing to up to 80% of all serious, preventable errors. To remedy this situation, a standardized handoff communication format is suggested. The other actions might work to some degree, but not to the extent that improving handoff communication would.

A child takes anti-inflammatory medications for a chronic condition. The mother asks the nurse why the child's cuts and scrapes seem to take so long to heal. Which response by the nurse is the most appropriate? A. "Chronic diseases always affect the healing process." B. "Inflammation is the first stage of healing wounds." C. "Sick kids tend to be anemic, which often delays healing." D. "Wound healing would not be affected in your child."

ANS: B The first stage of healing consists of the inflammatory process, which is being hampered by this child's need to take chronic anti-inflammatory medications. Although chronic disease can affect healing, this answer is too vague to be useful. Not all sick children are anemic. Wound healing definitely would be affected in this child.

23. A new nurse in the emergency department is examining a 4-year-old child who is sitting upright, is drooling, and is restless. Which action by the new nurse causes an experienced nurse to intervene? A. Attaches a cardiac monitor and oximeter B. Attempts to assess throat with a tongue blade C. Permits child to remain in an upright position D. Prepares to administer racemic epinephrine (MicroNefrin)

ANS: B This child has manifestations of acute epiglottitis. The nurse should not look into the child's throat, as this may precipitate laryngeal spasm and total airway obstruction. The experienced nurse should intervene if the new nurse attempts to do so. The other interventions are appropriate.

The pediatric clinic nurse assesses small, pink "pearl-like" lesions on the trunk of a school-aged child. Which treatment regimen does the nurse plan to teach the family about based on the assessment findings? A. Antibiotics twice a day for 7-10 days B. Cleaning the bathtub after the child bathes C. Intramuscular injection of an antibiotic D. Topical use of tretinoin (RetinA) on lesions

ANS: B This child has manifestations of molluscum contagiosum, a viral infection that generally resolves on its own. Because this is contagious, the bathtub should be disinfected after the child bathes and his or her towels should not be shared. More aggressive treatment is only used for cases that do not resolve or when the lesions become infected.

A nurse has been asked to perform a HEENT assessment on a child. What areas does the nurse assess? (Select all that apply.) A. Endocrine B. Eyes C. Head D. Nose E. Throat

ANS: B, C, D, E HEENT stands for head, eyes, ears, nose, and throat. Endocrine is not included.

A toddler has been adopted from a foreign country and has the nursing diagnosis of delayed growth and development related to poor nutrition. Which activities should the nurse include in the plan of care to assist the child to meet outcomes? (Select all that apply.) A. Allow the toddler to use a fork and knife with supervision. B. Allow toddler to choose foods within appropriate parameters. C. Consult with a dietician to provide a high-calorie, high-protein diet. D. Force the toddler to eat if he or she chooses not to at mealtime. E. Provide only foods that the toddler can eat with the fingers.

ANS: B, C At the toddler stage, the child can use a cup and a spoon and is able to verbalize food preferences. A dietary consult is appropriate to help plan high-protein, high-calorie meals and snacks that are preferred by the child. A toddler is not dexterous enough for a knife and fork. Eating should not be forced. Finger foods are a good choice for eating, but in the toddler stage, foods should not be limited to only finger foods because the child should be able to use a spoon.

A mother brings her 8-year-old son into the clinic for a well-child visit. She is concerned because her son does not take responsibility for completing and handing in his school assignments on time, and his teacher has given him several warnings for missing homework. Which responses are appropriate for the nurse to this child's mother? (Select all that apply.) A. "A good discipline strategy for this stage of development is a time-out in a specific location, which could include focusing on homework at this time." B. "As a parent, you should refrain from rescuing your child from the consequences of his behaviors and allow him to learn a valuable lesson from them." C. "At 8, children can internalize rules, so it's important to allow him to be responsible for his actions and to accept the consequences of his behavior." D. "Eight-year-old children are not ready to accept the consequences of their actions and need to be constantly reminded of their responsibilities." E. "Natural consequences are important, and removing privileges, especially time with friends, is often a successful technique at this age."

ANS: B, C Because the child in this stage of development is beginning to internalize rules, it is important to allow the child more independence and, thus, more awareness of the natural consequences of behavior. An effective parenting technique is to refrain from "rescuing" the child from the consequences of his or her behavior. Time-outs are more appropriate for preschool-age children. Grounding or restricting privileges is a more appropriate discipline measure for a school-age child, and restricting time with friends is a good technique for adolescents.

A 2-year-old child has croup. Which nursing interventions are appropriate for this child? (Select all that apply.) A. Administering oral or intravenous antibiotics B. Administering racemic epinephrine (Micronefrin) C. Applying oxygen via an oxygen tent D. Teaching the parents how to give cough medication E. Using a warm-mist humidifier

ANS: B, C Commonly, mild cases of croup are treated with cool mist. In the hospital setting, oxygen hoods for infants and oxygen tents for toddlers are used. Nebulized racemic epinephrine (Adrenalin) and l-epinephrine are equally effective in inducing mucosal vasoconstriction, with a consequent decrease in subglottic edema, thus relieving the symptoms. Antibiotics are not used in the management of croup, as it is primarily viral in nature. Cool-mist humidifiers should be used rather than warm mist due to the potential for bacterial growth in the warm-mist system.

A nurse is providing a child safety class to a parent group. Which instructions are appropriate for the age group? (Select all that apply.) A. Adolescent: Ride in the back seat of the car. B. Infant: Never attach a pacifier to the infant's clothing. C. Preschool: Teach stop, drop, and roll for fire. D. School age: Keep toilet seats down and tubs empty. E. Toddler: Never leave unattended in a walker.

ANS: B, C For an infant, parents should be taught not to attach a pacifier to the infant or infant's clothing. Walkers are not recommended, but were used on infants in the past. Preschoolers can learn fire safety, including stop, drop, and roll. Keeping toilet lids down and bathtubs empty is appropriate for toddlers. School-age children should ride in the back seat of a car.

The pediatric nurse explains to a student that which actions are most important in preventing and controlling infections? (Select all that apply.) A. Administering antibiotics B. Educating the public C. Monitoring for outbreaks D. Providing immunizations E. Scheduling physical exams

ANS: B, C, D Prevention and control of infections, especially communicable diseases, centers around surveillance, public education, and immunization.

Parents of a 3-year-old call the clinic and ask for information on sinusitis. Which symptoms will the nurse include in the explanation to the parents? (Select all that apply.) A. "Cold" lasting more than 5 days B. Cough worse when lying down C. Periorbital swelling D. Postnasal drip E. Thin, clear nasal discharge.

ANS: B, C, D The signs and symptoms of sinusitis include a cold lasting more than 10-14 days, sometimes with low-grade fever, thick yellow-green nasal discharge, and postnasal drip leading to sore throat; cough (worse at night or when lying down); bad breath; nausea and vomiting; headaches; irritability and fatigue; and swelling around eyes.

A school nurse is working with high school students. The nurse wants to influence these teenagers to make healthy decisions. Which actions are most appropriate by the school nurse to influence health decisions? (Select all that apply.) A. Discouraging questioning rules B. Discussing without judging C. Encouraging critical thought D. Listening carefully to the teen E. Paying attention to nonverbal cues

ANS: B, C, D, E A teenager is able to think abstractly and can be led to think critically and solve problems. At this level of emotional development an adolescent can take responsibility for his or her own actions and is beginning to internalize a personal set of values. Therefore, guiding them, but being an open communicator, is appropriate per their development. Adolescence is a time of great questioning, and this should not be discouraged.

The pediatric nurse describes the effects of cystic fibrosis on the body systems to the parents of a child recently diagnosed with the disease. Which statements does the nurse include to the parents? (Select all that apply.) A. Altered protein and vitamin metabolism causes a type of dementia in older children. B. Increased mucus obstructs the airways, and stasis of fluid causes infections. C. Pancreatic ducts are often blocked by mucus, leading to poor nutrition. D. Reproduction is affected, as ovarian ducts and the vas deferens are occluded. E. Thick mucus affects several body systems, preventing some organs from working.

ANS: B, C, D, E Cystic fibrosis is an inherited autosomal recessive disorder that causes the production of thick mucus that blocks exocrine glands and affects several body systems, including the respiratory, gastrointestinal, and reproductive systems. It does not lead to a type of dementia.

The nurse conducts a seminar for new nurses about child safety in the hospital setting. Which of the following should be included? (Select all that apply.) A. Asking children to state their names to ensure safe medication administration B. Basing safety measures on the developmental level of the children C. Keeping play areas in the hospital open at all times for children to visit D. Providing age-appropriate transportation methods to other areas of the hospital E. Storing toxic and nontoxic materials on the top shelf of a locked cabinet

ANS: B, D, E Safety measures instituted on a pediatric unit are based on the developmental level of children to protect them from harm. Safety measures include keeping toxic materials out of reach, identifying children with name bands, and knowing the whereabouts of children on the unit, including in the play areas. Providing safe, age-appropriate transportation is also important.

The nurse is preparing to educate the parents of an 8-year-old child about normal growth and development. Which information should the nurse include? (Select all that apply.) A. Boys and girls play equally with each other. B. Children frequently have best friends at this age. C. Peer approval is not yet important, but will be for teens. D. Puberty changes should be discussed before they occur. E. Typical weight gain is 4-6 lb (1.8-2.7 kg)/year.

ANS: B, D, E School-age children typically gain 4-6 lb/year. This is an appropriate time to begin discussing changes that occur during puberty because those changes can be frightening and the child should be prepared for them. Friends, especially best friends, which become apparent at this age, are usually of the same gender. Peer approval is becoming increasingly important at this stage.

A nurse is educating a community group of parents about prevention of West Nile virus. Which information does the nurse include in the teaching session? (Select all that apply.) A. All children should be sprayed with DEET before going outside. B. Eliminate standing water around your house, such as in birdbaths. C. Holistic mosquito repellent, such as lavender, is very effective. D. Long sleeves and long pants help prevent mosquito bites. E. The peak season for this virus is late summer to early autumn.

ANS: B, D, E West Nile is most prevalent in late summer and early fall. Standing water is a breeding ground for mosquitoes and should be eliminated. Long sleeves and pant legs help keep mosquito bites from occurring on the arms and legs. Mosquito repellants containing DEET are most effective in preventing mosquito bites. Holistic methods are not as effective. Children under the age of 2 years should not have DEET sprayed onto their skin; rather, it should be applied to their clothing.

What does the pediatric nurse understand about the use of complementary and alternative medicine (CAM)? (Select all that apply.) A. All CAM practices are the same within a cultural group. B. CAM practices reflect ethnocultural health beliefs. C. Only herbal remedies are considered CAM therapies. D. Over half of the adults in America use CAM. E. Very few doctors collaborate with CAM providers.

ANS: B, E CAM practices reflect ethnocultural beliefs (relating to a particular ethnic group), and a culturally competent nurse will take the time to become familiar with CAM practices in the diverse communities served. According to one study, only 4% of pediatricians communicate with CAM providers or initiate the recommendations of the CAM provider. All members of the same cultural group may not adhere to the same CAM practices (although many of them will), and the nurse should not stereotype members of the group and assume they do. CAM consists of multiple modalities such as herbal therapy, massage, biofeedback, and yoga. More than a third of adults in America use CAM therapies.

A nurse is working at a community health-screening event. A woman tells the nurse that her oldest child is leaving home in a month and that this change is making her feel stressed and unneeded. What response by the nurse is the most appropriate? A. "At your age you need to start coping with losses." B. "Lucky you; I can't wait 'til my kids leave home!" C. "This is a great time to refocus on your marriage." D. "Your other children will need help with separating."

ANS: C According to Duvall's theory of family development, the activities of the oldest child in a family are the marker for family transition. As the oldest child prepares to leave home (family launching young adult stage), the marriage needs to become a major area for focus. This activity starts in the stage in which the oldest child is an adolescent and looking to launch in the future. The role of the older family is to start coping with loss. The statement that the woman is lucky is disrespectful and does not address her needs. Helping children with separation is the task of the family with preschool children.

A nurse notes that when an infant is startled, she looks at her mother. What conclusion can the nurse make about this infant's development? A. The child is slow to adapt and is distressed over small changes. B. The developmental needs of the child are not being met. C. The infant can develop other relationships because he is secure. D. The infant has an unstable home environment and is insecure.

ANS: C According to John Bowlby's theory of attachment, the infant becomes attached to the mother as a way to survive the vulnerabilities of infancy. When the attachment is secure, the mother is seen as home base. When the child becomes startled or frightened, he or she will look to the mother for security. Knowing that home base is secure allows the child to go on to develop other relationships. The other statements are not accurate.

A father is frustrated that his child frequently disobeys well-established rules and then attempts to excuse the action by stating "Well I just thought . . . ." The father asks for guidance with discipline. What information is most appropriate for the nurse to provide? A. Ensure the child knows the rules. B. Habitual rule breaking is problematic. C. This is a normal developmental stage. D. Time-outs are ineffective for lying.

ANS: C According to Kohlberg, a child passes through three stages when learning to make moral decisions. In the first (preconventional) stage, a child may not see an action as wrong if he or she can justify the action in his or her mind. The best response by the nurse would be to explain this to the father and help him develop strategies to work with this. Ensuring the child knows the rules is important before inflicting consequences for behavior. Stating that rule breaking is problematic does not help the father deal with the problem. Time-outs can be effective disciplinary strategies.

9. A pediatric clinic nurse teaches parents how to care for their toddler who has nasal congestion. What anatomical difference between children and adults is a concern with congestion in children? A. Children this age should not have congestion. B. Larger tonsils trap mucus, leading to gagging. C. The narrow trachea can become obstructed easily. D. Phlegm can migrate into the eustachian tubes.

ANS: C Excess mucus production can lead to airway obstruction in children due to the narrowed lumen size of their tracheas.

30. A 5-month-old child is brought to the pediatric clinic by the parent, who reports the child has had a cough for 4 weeks. When reviewing the child's history, the nurse assesses that the child's last DTaP vaccination was at 2 months of age. Which action by the nurse is the most appropriate? A. Inform the provider that both child and parent need antibiotics. B. Make an appointment for the next vaccination at 6 months. C. Obtain nasal washings for a culture and PCR testing. D. Weigh and measure the child; document all findings in the chart.

ANS: C All children with a chronic cough lasting longer than 3 weeks should have diagnostic testing for pertussis, including nasal washings for culture and PCR tests. This child has the risk factor for pertussis of inadequate immunizations; he should have had a second vaccination at 4 months. The best action by the nurse is to collect the specimens. If positive, both child and caretakers will need antibiotics, as pertussis is highly contagious. The child should receive a booster shot as soon as he is recovered. All children should be weighed and measured and findings documented in the chart.

Which outcome is most appropriate for a 3-year-old with the nursing diagnosis of delayed growth and development related to chronic illness? A. Child will attain age-appropriate milestones in 3 months. B. Child will attain normal weight and height in 2 months. C. Child will draw using large crayons in 2 months. D. Child will have fewer tantrums in 3 months.

ANS: C All outcomes might be good starts, but an outcome needs to be measurable, be specific, and have a timeframe. The outcome that best fits this description is the one that has a specific age-appropriate activity associated with it. "Attain age-appropriate milestones," "attain normal weight and height," and "fewer tantrums" are all vague.

20. A patient is experiencing an anaphylactic reaction. Which action by the nurse takes priority? A. Determine what the patient is allergic to. B. Listen to the patient's lung sounds. C. Maintain the patient's airway. D. Provide oxygen at 4 L nasal cannula.

ANS: C Anaphylaxis is a medical emergency. Airway comes first. The patient may need oxygen, but if the airway is not patent, the oxygen will not help. Listening to the lungs and determining the allergen come later.

31. A child is being sent home from the doctor's office with a prescription for azithromycin (Zithromax) for presumed cat-scratch disease. Which instruction to the parents is most important? A. "Be sure to treat your cat for fleas." B. "Don't take this unless the scratch gets infected." C. "Make sure he takes all of this antibiotic." D. "You should not have cats around small children."

ANS: C As with any antibiotic, taking all the prescribed medication is a priority instruction. For some reason, cats with fleas have higher rates of the bacteria that causes the disease, so flea control is important. The other two instructions are not appropriate.

A child has blood testing for possible atopic dermatitis. The parent asks why testing for IgE is done. Which response by the nurse is the most appropriate? A. "High levels of IgE from an overactive immune system are treated with steroids." B. "If we can control the levels of IgE, we can control your child's skin condition." C. "IgE is raised in allergies and asthma, which are often seen with atopic dermatitis." D. "These results will help us determine how your child's immune system is functioning."

ANS: C Atopic dermatitis is often seen in children with allergies and asthma. In those conditions, levels of IgE are elevated, so this correlation helps confirm the diagnosis if it is in question. The other answers are inaccurate.

A mother-baby nurse is demonstrating swaddling a neonate as a method of keeping the baby warm. The mother asks why she needs to be so careful about the baby's temperature. Which response by the nurse is the most appropriate? A. "After being in your womb for 9 months, this is how the baby prefers to stay." B. "Babies' immune systems are immature; any cold breeze will make them sick." C. "Their skin is thin and they have little fat, so it's hard to control their temperature." D. "You need to keep their temperature above what we call normal to keep them well."

ANS: C Babies have thin skin, a large body surface area, scanty subcutaneous fat, and immature neurological systems. All these factors combined make it difficult for a baby to regulate his or her temperature. The other statements are not factual or informative.

28. A child is admitted to the pediatric unit with respiratory syncytial virus (RSV). Which action by the nurse is best for infection control? A. Adhere to policy on hand hygiene. B. Do not assign pregnant caregivers. C. Place the child in contact isolation. D. Use meticulous standard precautions.

ANS: C Children with RSV are placed in contact isolation due to the ease of spreading the disease. Of course nurses should adhere to facility policy on hand washing, but further action is required. There is no precaution against pregnant caregivers. Standard precautions are used on all patients, but a child with RSV needs contact isolation.

6. A 10-month-old-child is in the pediatric clinic for his eighth ear infection. Which assessment is most important for the nurse to perform on this child? A. Ask the parent about possible allergy testing. B. Assess the child's mouth for oral thrush. C. Graph height and weight on the growth chart. D. Inquire about the health of the entire family.

ANS: C Children with primary immunodeficiencies can often be identified using Modell's 10 Warning Signs, which include failure to gain weight or grow properly. The nurse should assess the child's height and weight and graph it on the growth chart, comparing it to normal values for the child's age. Persistent oral or skin thrush is another sign if it persists past 1 year of age. Assessing for parental views on allergy testing is not related. Because these deficiencies are congenital, asking about the health of the entire family is too vague; it would be important to ask specifically about a history of primary immunodeficiencies, however.

22. A parent calls a pediatric information line to ask about treating sinus congestion in a child. Which suggestion is not appropriate for the nurse to make to the parent? A. Warm facial compress B. Cool-mist steamer C. Sine-Off sinus medication D. Gentle nasal suctioning

ANS: C Common comfort measures for sinus or respiratory problems include a cool-mist steamer, decongestants, and gentle nasal suctioning. A warm facial compress would be more helpful. Sine-Off over-the-counter sinus medication contains salicylates, or aspirin compounds, which are not given to children due to the risk of Reye's syndrome.

When teaching a parenting group how to prevent the most common cause of childhood burns, what action does the nurse advise? A. Cover electrical outlets. B. Keep children out of the kitchen. C. Install smoke detectors. D. Lock up household chemicals.

ANS: C The most common type of childhood burn is a thermal burn. Installing smoke detectors provides early warning and the chance to escape a fire.

A toddler is brought to the clinic with a low-grade fever and the mother describes a grunting sound made by the child on expiration. The respiratory rate is 24 breaths/minute. What action by the nurse is most appropriate? A. Assess nose and throat for foreign bodies. B. Facilitate a stat chest x-ray. C. Obtain an oxygen saturation; notify provider. D. Weigh and measure child then calculate BMI.

ANS: C Grunting noises are heard at the end of expiration and are caused by glottal closing. They can be indicative of respiratory distress or pneumonia. This child's respiratory rate is normal for age. The nurse should perform further assessments and notify the provider. Grunting is not associated with foreign bodies. The child is stable and does not need a stat chest x-ray, although he or she probably will have one after being seen. The BMI might be important for a child who snores or breathes noisily because snoring is often associated with obesity (along with foreign body, nasal polyps, choanal obstruction, or hypertrophied adenoid tissue).

40. The pediatric charge nurse receives this report on an incoming admission: a 3-year-old boy with ear and jaw pain, bilateral parotid gland swelling, and mild dehydration. Which action by the charge nurse is most appropriate? A. Do not assign pregnant nursing staff. B. Inform parents that sterility is common. C. Place the child on droplet precautions. D. Place the child on airborne precautions.

ANS: C Hospitalized children who have mumps require droplet precautions. There is no danger to a fetus. Sterility is possible in male children due to orchitis, but it is rare.

A mother takes her 10-year-old son to the pediatrician for a sprained wrist. During the medical history, the pediatric nurse listens to the mother describe her son as "busy playing basketball all day long with the other boys in the neighborhood." Based on the nurse's assessment, which stage of Freud's psychosocial development is this child experiencing? A. Anal stage B. Genital stage C. Latency stage D. Phallic stage

ANS: C In the latency stage (6 to 12 years) Freud believed that the child "takes a break" psychosexually. This allows the child to focus more intently on other aspects of growth and learning, such as spending time with same-gender friends or excelling in sports or video games. At this age, the child presumably has little interest in issues of sexuality. The oral stage occurs from birth to 1 year, and the infant is fascinated with oral curiosity. The genital stage occurs between 12 and 18, and sexuality and relationships are the focus. The phallic stage occurs from 3 to 6 years, during which sexual differences are discovered.

A nurse is assessing a school-age child who complains of stomach aches after eating. Which question is appropriate for the "D" component of the OLD CAT mnemonic? A. "Can you describe how your tummy pain feels?" B. "Have you tried any over-the-counter drugs?" C. "How long does the pain last after you eat?" D. "What day did you first notice the pain?"

ANS: C OLD CAT stands for onset, location, duration, character, aggravating/alleviating factors, and timing. Asking the child how long the pain lasts reflects duration.

A parent calls the pediatric clinic asking for advice on treating lice. The child has already been treated once with lindane (Qwell). Which advice from the nurse is the most appropriate? A. "Be sure you are cleaning your house and linens well." B. "Qwell is usually effective after two or three treatments." C. "Switch to malathion (Ovide) and see if that works better." D. "There are some oral medications your child can try."

ANS: C Ovide is a safe, nontoxic lice shampoo that is very efficacious. The mother should not be told to use Qwell again; the FDA has a black box warning against using this as a first-line drug and the American Academy of Pediatrics no longer recommends it at all. There are oral medications that can be used on resistant lice, but the parent needs to try another shampoo first. Cleaning is important, but not as important as treating the lice.

A nurse reads in a child's chart that the child has pectus carinatum. What does the nurse understand this term to mean? A. Barrel chest from chronic illness B. Depression of the lower chest C. Protrusion of the chest D. Underdeveloped breast bone

ANS: C Pectus carinatum is an abnormal protrusion of the chest. Depression of the lower portion of the sternum is known as pectus excavatum.

49. A nurse is assessing a 10-year-old child with asthma. The child and parents report daily symptoms, but the child only wakes up coughing once or twice a week, uses an albuterol (Ventolin) inhaler daily, and is occasionally prevented from participating in physical activity by symptoms. What asthma severity classification does the nurse assign to the child? A. Intermittent B. Mild persistent C. Moderate persistent D. Severe persistent

ANS: C The child's symptoms place him or her in the moderate persistent category.

19. A patient is experiencing an anaphylactic reaction. Which IV solution does the nurse anticipate will be ordered for this patient? A. 0.45% normal saline B. 5% dextrose in water C. 0.9% normal saline D. 3% saline

ANS: C The fluid of choice in any emergency is an isotonic crystalloid; 0.9% normal saline is isotonic, 0.45% normal saline and 5% dextrose in water are both hypotonic, and 3% saline is hypertonic.

A visiting nurse is making a home visit on a male 2-month-old child who was born prematurely. The nurse notes that the child has not been circumcised. What action by the nurse is most important? A. Assess the number of the baby's wet diapers per day. B. Give parents a referral to have the child circumcised. C. Instruct parents not to retract the foreskin until after age 1. D. Teach parents to retract the foreskin for cleaning.

ANS: C The foreskin of an uncircumcised male baby should not be forcibly retracted until after he is 1 year of age. Assessing the number of wet diapers per day is important for any infant and is not directly related to the uncircumcised baby. The nurse should not just give a referral for circumcision, as the parents have chosen the option of not circumcising their baby, which may be for personal, religious, or cultural reasons. After 1 year of age, the foreskin will need to be retracted for cleaning. It should be returned to its original position afterward.

48. A 1-year-old child has the following arterial blood gas values (ABGs): pH: 7.28, PCO2: 58 mm Hg, PO2: 77 mm Hg, HCO3: 14 mEq/L, O2 saturation: 88%. Which interpretation of the results by the nurse is the most accurate? A. Metabolic acidosis B. Metabolic alkalosis C. Respiratory acidosis D. Respiratory alkalosis

ANS: C The low pH indicates acidosis; the high CO2 level indicates respiratory involvement. The child is also hypoxemic.

A nurse is providing anticipatory guidance to the parents of a toddler. Which information is most important for the nurse to include regarding typical toddler behaviors? A. Children at this age rarely lie about misbehaving and want to please their parents. B. Competing and winning are important, so "rewards" for good behavior are great tools. C. Toddlers need constant supervision, because they don't understand what consequences can occur. D. Time-out for misbehavior works well if the child is placed quietly in his or her own room.

ANS: C The toddler is actively exploring his or her world, and with this increasing sense of mastery and lack of knowledge of consequences, toddlers often end up in situations in which their safety is compromised. Keeping the child safe becomes increasingly challenging. At this point language is developing and is focused on the child's needs. Competing and winning are important in the early childhood years (ages 6-9). Locations for time-outs need to be safe, which means in a location the parent can monitor and see.

The pediatric nurse takes a comprehensive health history of a 10-year-old patient and asks the parents about their use of herbal products or home remedies. What information does the nurse know regarding herbal products? A. Aloe vera can affect clotting time by decreasing platelets. B. Bilberry can cause hypersensitivity in patients with allergies to plants. C. Echinacea is contraindicated for patients with autoimmune disorders. D. Fennel is contraindicated in patients with diabetes, hypertension, or liver disease.

ANS: C There are no known side effects for topical application of aloe vera. Bilberry can affect clotting time by decreasing platelet aggregation. Echinacea should not be used for patients with autoimmune disorders, diabetes, AIDS, or HIV. Fennel may have a laxative effect; licorice is contraindicated in patients with diabetes, hypertension, or liver and kidney disease.

A hospitalized 11-year-old child turns down opportunities to play or engage in diversionary activities. When questioned, the child states "I'm bad at that" or "I can't do anything." What action by the nurse is best? A. Arrange a pediatric psychology consultation. B. Assess the child for emotional abuse at home. C. Consult the child developmental specialist. D. Document the statements in the child's chart.

ANS: C This 11-year-old child is in the Erikson stage of industry versus inferiority, and it seems he or she has not mastered tasks and developed a sense of self-confidence. Illness can frequently disrupt growth and behavior, and the child developmental specialist is a vital resource in meeting the developmental needs of the hospitalized child. Documentation should always be accurate and thorough, but simply documenting the statements will not help resolve the problem. The child may or may not need a psychology consultation or an abuse assessment, but the focus is on helping the child meet developmental milestones.

A mother brings her 1-year-old child to the pediatric clinic and appears frustrated and stressed. During the assessment, the mother states she tries to give her child exposure to new situations and people several times a week, but the outings always end with the child screaming and crying. Which response by the nurse is the most appropriate? A. "Keep trying; new situations are so stimulating for children." B. "Stop taking your child to new places and meeting new people." C. "Use an established routine and add new experiences slowly." D. "Your child will soon become used to such daily activity."

ANS: C This child displays difficulty with adapting to new situations. The mother's attempts to provide new experiences are antagonizing the child's natural temperament. According to the temperament theory of Thomas, Chess, and Birch, the mother should provide structure with limited variation in this slow-to-adapt child's daily activities.

A school-age child with asthma came to the emergency department with a respiratory rate of 44 breaths/minute and wheezes heard throughout. After two breathing treatments, the nurse assesses a respiratory rate of 8 breaths/minute and hears no wheezing. The child is lying quietly on the bed. What action by the nurse is best? A. Allow the child to rest undisturbed. B. Call for another respiratory treatment. C. Obtain oxygen saturation; notify provider. D. Reassess the child in 30 minutes.

ANS: C This respiratory rate is too low for a child of any age and is indicative of exhaustion and the inability to breathe effectively. The absence of wheezes may indicate lack of ventilation. The nurse should obtain an oxygen saturation and notify the provider immediately. Without action, the child could progress to respiratory arrest. There is no indication that the child needs another breathing treatment.

A nurse is weighing a 2-month-old infant in the clinic. To ensure safety, which action is most appropriate? A. Have the parent hold the child while standing on an adult scale. B. Place the baby in the scale and place one hand on top of the baby. C. Place the baby in the scale and hold one hand just over the baby. D. Prop the infant sitting up in the scale, then weigh the prop separately.

ANS: C To protect a child from an accidental fall from the infant scale, the nurse places the baby in the scale and holds one hand just over the baby. Weighing the adult and baby, then subtracting the adult's weight is not as accurate as using the infant scale, which reads in smaller increments. Placing a hand on top of the baby will add weight. Propping up a 2-month old-infant is not as safe as laying the infant down.

The pediatric nurse working in a hospital setting uses both standard precautions and transmission-based precautions for patients. Which patient requires only standard precautions? A. Infectious diarrhea B. Staphylococcal infection C. Tonsillitis D. Tuberculosis

ANS: C Transmission-based precautions are intended to prevent the transmission of pathogens from those with infectious diseases. Transmission-based precautions include airborne, droplet (TB), and contact precautions (infectious diarrhea and staph infection). Standard precautions are used on all patients, including those with tonsillitis.

2. The pediatric nurse explains to the parents of a 1-year-old patient with pneumonia that the differences between the adult's and child's respiratory system affect function and subsequent respiratory conditions. Which difference does the nurse include in the discussion with the patient's parents? A. Infants are obligate nose breathers until 6 months of age. B. The epiglottis in the child under 8 is shorter and more rigid. C. The larynx and the glottis are higher in the younger child's neck. D. In the child, there are more functional muscles in the neck and less soft tissue.

ANS: C Until about age 4 weeks, infants are obligate nose breathers and do not open their mouths to breathe. The epiglottis in the younger (usually age 8 years and younger) child is longer and flaccid (floppy), which makes it more susceptible to swelling. The larynx and the glottis are higher in the younger child's neck, which makes the child more prone to aspiration. There are fewer functional muscles in the neck, and the increased amount of soft tissue makes the younger child more susceptible to infection and edema.

A nurse working on a pediatric unit teaches a student nurse that children may require intravenous fluids to be administered through one of several different venous access devices. The nurse tells the student that which of the following current patients are candidates for central venous access devices? (Select all that apply.) A. Receiving blood product replacement after surgery B. Receiving fluid maintenance prior to diagnostic testing C. Requires long-term intravenous access for hydration D. Will need regularly scheduled chemotherapy E. With medication administration scheduled preoperatively

ANS: C, D A peripheral line is used for short-term intravenous therapy, including transfusions, the possibility of future intravenous therapy, or intermittent medication administration. Those children with a condition necessitating long-term intravenous access are candidates for central venous access devices. The children who will need long-term intravenous hydration or chemotherapy are candidates for a central venous access device.

The nurse is assessing a patient diagnosed with cystic fibrosis. Which findings support the patient's diagnosis? (Select all that apply.) A. Concave chest B. Dry, scaly skin C. Protuberant abdomen D. Wasted buttocks E. Thick extremities

ANS: C, D Protuberant abdomen, barrel chest, wasted buttocks, and thin extremities are common features in children with cystic fibrosis.

The pediatric nurse caring for children with physical disabilities teaches the student nurse that coping mechanisms give rise to resiliency in children and their families. The nurse further notes that attributes of resilient children include which of the following? (Select all that apply.) A. Dependence on family B. Physical abilities C. Problem-solving skills D. Sense of purpose and future E. Social competence

ANS: C, D, E Four common attributes of resilient children are social competence, problem-solving skills, autonomy, and a sense of purpose and future. Dependency and degree of impairment are not related.

A child with pertussis is in the catarrhal stage of the disease. Which assessment findings correlate with this condition? (Select all that apply.) A. Chronic cough lasting weeks B. Intense cough causing vomiting C. Low-grade fever D. Sweating and fatigue after coughing E. Upper respiratory symptoms

ANS: C, E The catarrhal stage of pertussis is characterized by upper respiratory symptoms and low-grade fever. Sweating, fatigue, and intense coughing with vomiting are characteristic of the paroxysmal stage. A chronic cough lasting weeks is seen in the convalescent stage.

36. A patient was hospitalized 2 years ago with a resistant bacterial infection. The patient is admitted for an unrelated problem and placed on contact isolation. The parents question the need for this action. Which response by the nurse is best? A. "It is possible that your child could still contaminate the nursing staff." B. "It's our policy to isolate anyone who has had this infection in the past." C. "This seems distressing for you; would you like me to call the charge nurse?" D. "Your child may be colonized with the bacteria so we isolate until we know."

ANS: D A person who had a bout with a resistant bacteria may be colonized. Many facilities require placing such patients in isolation until this has been ruled out. This is the most factual and informative answer. Contaminating the nursing staff is not really the problem; the nursing staff spreading the organism to susceptible patients is. The nurse should provide information and not just call someone else to explain. Telling the parents that this practice is policy may be true, but does not give them any information.

A nurse is explaining to a nursing student that a patient experienced a sentinel event during a previous hospitalization. What does the student understand about this event? A. Experienced an unusual event that is rare in the literature B. Had an unexpected response to treatment or nursing care C. Meeting a major milestone in treatment for an illness D. Unexpected event resulting in serious injury (or death)

ANS: D A sentinel event is an unexpected event that results in the death or serious injury of a patient. The other descriptions are inaccurate.

20. A 5-year-old child is brought into the clinic by a parent, who reports the child has a "sore throat." Which assessment finding would require immediate notification to the health-care provider? A. Difficulty swallowing B. Inflamed, red pharynx C. Refusing to eat the last 2 days D. Strawberry-colored tongue

ANS: D A strawberry-red tongue, petechiae on the palate, and a fine red rash on the trunk or abdomen are consistent with pharyngitis caused by Streptococcus A infection, which needs immediate treatment. The other manifestations are seen with viral pharyngitis infections.

A 2-year-old is seen for a well-child visit and is scheduled to receive immunizations. The child weighed 22 lb (9.97 kg) at 1 year of age (1 year ago). Today the child weighs 23 lb (10.4 kg). Which conclusion is most appropriate for the nurse to make regarding this assessment data? A. The child is at an expected weight. B. The child is over expected weight. C. The child is seriously overweight. D. The child is underweight for age.

ANS: D A toddler should gain 4-6 lb each year from the ages of 1 to 3. This child should now weigh somewhere between 26 and 28 lb, so he or she is underweight.

A nurse is working with a child at a nutrition site. The father is not in the child's life, and the mother has been in and out of jail, resulting in a series of caregivers for the child, who appears malnourished. Using Bandura's theory of growth and development, what should the nurse assess as a priority? A. Bonding B. Industry C. School success D. Self-esteem

ANS: D According to Bandura, lack of positive modeling leads to poor self-esteem, and the child has no opportunity to master developmental tasks and skills. Bonding, industry, and school success are not directly related to Bandura's theory, although correlations can be made.

47. A nurse is using the CUPS method of assessing a child's respiratory status and documents the following: open airway containing secretions, increased respiratory rate and effort, wheezing, mucous membranes pale pink in color, and fever. Based on this system, which is the appropriate rating for this child? A. Critical B. Potentially unstable C. Stable D. Unstable

ANS: D According to this scale, the child has four determinants of unstable status (increased breathing rate, increased breathing effort, wheezing, and pale pink mucous membranes). An open airway with secretions and presence of fever both characterize the potentially unstable category. The nurse would classify this child as unstable.

A father brings his 10-month-old infant to the well-baby clinic and expresses frustration that his child "puts everything in her mouth" and "gets into everything," then cries when objects are taken away. In providing anticipatory guidance, what action by the nurse is the most appropriate? A. Advise the father that crying indicates fatigue and the child should rest. B. Explain that objects may be dirty and should not be put in the mouth. C. Give the father written material on effective disciplinary techniques. D. Teach the father that this behavior is normal and not misbehaving.

ANS: D At this age, exploration and crying are normal behaviors. Children in this age group do not misbehave, and so discipline needs to be focused on redirection to keep the child safe. Crying may also indicate frustration in this preverbal child. Making a blanket statement that objects are dirty and should not be in the child's mouth does not provide guidance on age-appropriate behaviors and reasonable responses.

A nurse is providing nutritional information to a parent group. Which information is most appropriate? A. At least 35% of calories should come from protein. B. Limit carbohydrates to 10-15% of daily calories. C. Saturated fats are the healthiest fat choice. D. Use whole milk until your child is 2 years old.

ANS: D Children should be switched from whole milk to skim or low-fat varieties after age 2 years. Ten to 35% of calories should be from protein sources. Unsaturated fats are healthier than saturated fats. Carbohydrates form the bulk of a child's diet and should be 45-65% of the daily calorie intake.

19. The mother of a 5-year-old child calls the clinic to ask if her child has a mild respiratory infection or needs to be seen. Which question by the nurse would elicit the most helpful information? A. "Can your child swallow without pain?" B. "Does the child have a sore throat?" C. "Is your child coughing occasionally?" D. "Was the onset gradual or sudden?"

ANS: D Common "colds" or nasopharyngitis usually have a gradual onset. A rapid onset would indicate a more serious condition. The other manifestations are commonly seen in this disorder.

On physical assessment of the skin of a patient, the nurse documents cyanosis. What other related assessment should the nurse perform? A. Ask the parent about yellow and orange vegetable intake. B. Draw blood for hemoglobin, hematocrit, and liver function studies. C. Palpate all the child's lymph nodes, assessing for enlargement. D. Take the child's vital signs, including blood pressure and pulse.

ANS: D Cyanosis may indicate a compromised cardiorespiratory state, and the nurse should assess measures of cardiac output and respiratory function. Taking vital signs will give the nurse information about these two systems. Vegetable intake, laboratory studies (including liver function tests), and palpating lymph nodes are not related to cyanosis.

A young couple brings their 20-month-old daughter to the pediatrician's office for immunizations. The mother tells the nurse that she is going back to work and is looking for a day-care center in the vicinity of the clinic. What assessment is priority before recommending a day-care center that would help the parent and child adapt to the experience? A. Available financial assets B. Available support people C. Potty-training status D. The child's temperament

ANS: D During a normal daily routine, the child may be exposed to a variety of settings and to several people in a day-care center or while visiting extended family, visiting a physician's office, or in public places in the community. Understanding an infant's temperament is essential in the care of the child to help both the parent and child adapt to these experiences. Although the other options are certainly considerations, they are not nearly as important as assessing the child's temperament.

The nurse is providing care to a newborn client and family during a well-baby checkup. Using Duvall's family development theory, which question is most appropriate for the nurse to ask? A. "Do you plan to have more children?" B. "How far apart are your closest children?" C. "How many children do you have?" D. "How old is your oldest child?"

ANS: D Duvall's family development theory uses the oldest child as the marker for transitions within the family. The nurse should inquire about the age of the oldest child. The other questions are not directly related to Duvall's theory.

A student nurse asks a pediatric nurse what cephalocaudal development means. Which response by the nurse is most accurate? A. Growth that occurs from bottom to top B. Growth that occurs from midline outward C. Growth that occurs from outside to midline D. Growth that occurs from top to bottom

ANS: D Growth in infants occurs in a cephalocaudal direction, that is, from head to toe (top to bottom). Growth that occurs from midline outward is termed proximodistal. Growth does not occur from bottom to top or from the periphery to the midline.

3. A nursing student is learning about the immune system. Which statement about immunoglobulins is correct? A. Adult levels of IgG are reached by the age of 6 months. B. Children are born with adult levels of IgA. C. IgE leads the body's attack against bacteria and viruses. D. IgM is the first type of antibody made in response to infection.

ANS: D IgM is the first antibody made in response to an infection. Adult levels of IgG are reached by 1 year of age. Children attain an adult level of IgA by 5 years of age. IgE is important in the response against fungus spores, animal dander, and pollen.

34. The mother of a child diagnosed with influenza in the emergency department last night calls the family practice clinic to ask if she can give the child store-brand "flu medicine." Which response by the nurse is the most appropriate? A. "If your child is that sick she should take Tamiflu." B. "No, your child should not be sick enough for medicine." C. "Yes, if it's labeled for children you can give it to her." D. "You must make sure it does not contain salicylates."

ANS: D Medications containing aspirin products (salicylates) should not be given to children with viral illnesses due to the correlation with Reye syndrome. If a parent is asking about an over-the-counter medication, the nurse should advise the parent to ensure it does not contain this compound. The other responses are not appropriate.

The pediatric nurse explains to a new mother that two factors, "nature" and "nurture," may influence the formation of her child's essence. Which factor among the following would be considered "nature"? A. Cultural aspects B. Parenting skills C. The era in which the child develops D. Traits inherent in the infant

ANS: D Nature describes the traits inherent in the infant—biologically imposed idiosyncratic factors that create what and how each person "is." Nurture, on the other hand, refers to the influence of external events like parenting received, culture, or the "times" in which a child lives.


Related study sets

PGA 3.0 - Level 1 Facility Management

View Set

Tensile Testing and Material Deformation

View Set

Sadlier Oxford's Vocabulary Workshop Level H Unit 4

View Set

Provincial and Territorial Governments

View Set